PDA

Archiv verlassen und diese Seite im Standarddesign anzeigen : Sind die Interpretationen der Quantenmechanik mehr als nur Geschichten?


Timm
16.06.19, 09:34
Die Interpretationen der QM erzählen Gegensätzliches über das Zustandekommen von Messresultaten.

Einigkeit besteht darin, daß die Messresultate den Vorhersagen der QM entsprechen.

Welchen wissenschaftlichen Fortschritt erhofft man sich von Forschung über Interpretationen der QM?

Hawkwind
16.06.19, 10:44
Die Interpretationen der QM erzählen Gegensätzliches über das Zustandekommen von Messresultaten.

Einigkeit besteht darin, daß die Messresultate den Vorhersagen der QM entsprechen.

Welchen wissenschaftlichen Fortschritt erhofft man sich von Forschung über Interpretationen der QM?

Das hatte ich mich auch schon gefragt: wenn es eh nur um "Metaphysik" (per Beobachtung ununterscheidbare "Theorien"), warum dann überhaupt forschen?

Es ist wohl so, dass die gegenwärtigen Standard-Interpretation einigen Einschränkungen unterliegen. So sei die Kopenhagener Interpretation für das Labor geeignet, aber eben nur für geschlossene Systeme anwendbar. Man fragt nach einer allgemeineren Beschreibung, die auch im Kontext von Kosmologie und Quantentheorie anwendbar ist.

Ein Beispiel sind die Untersuchungen zur "consistent histories"-Interpretation:
https://link.springer.com/article/10.1007/BF02183396

Timm
16.06.19, 16:34
.

Ein Beispiel sind die Untersuchungen zur "consistent histories"-Interpretation:
https://link.springer.com/article/10.1007/BF02183396
“Consistent Histories” scheint über Kopenhagen hinaus zu gehen und das ohne den Kollaps der Wellenfunktion, in diesem Sinne ähnlich wie Neumaier’s “Thermal Interpretation”. Letztere beansprucht die Unitarität nicht zu verletzen. Ob das auch für CH gilt weiß ich nicht.

Eigentlich sollten Interpretation symphatisch sein, die ohne Viele Welten und ohne Kollaps unter Wahrung der Unitarität mit dem Formalismus der Quantenmechanik konsistent sind. Und doch, was bringt’s? Es muß ja mehr als “nur” eine intellektuelle Herausforderung sein, wenn sich hochangesehene Leute damit beschäftigen, oder?

Mirko Buschhorn
19.06.21, 11:04
Die meisten Messresultate entsprechen den Vorhersagen der QM, aber eben nicht alle. Warum beobachten wir mehr Materie als Antimaterie im Universum? Sollte es sie nicht in gleichen Mengen geben? Ich denke, es lohnt sich über die Interpretation der QM den Kopf zu zerbrechen.

TomS
22.06.21, 05:31
Einigkeit besteht darin, daß die Messresultate den Vorhersagen der QM entsprechen.

Welchen wissenschaftlichen Fortschritt erhofft man sich von Forschung über Interpretationen der QM?
Man erhofft sich eine umfassende und konsistente Beschreibung der Vorgänge einschließlich des Messprozesses mittels quantenmechanischer Modelle.

Eigentlich sollten Interpretation symphatisch sein, die ohne Viele Welten und ohne Kollaps unter Wahrung der Unitarität mit dem Formalismus der Quantenmechanik konsistent sind.
Diese beiden Themen spielen eine völlig unterschiedliche Rolle.

Während das Kollapspostulat - auch wenn das einige Physiker immer noch nicht sehen wollen - aus instrumentalistischer Sicht verzichtbar ist, handelt es sich bei den Vielen Welten um eine elementare mathematische Vorhersage der Theorie. Der Anspruch ist also jeweils ein anderer: Im Falle des Kollapses liegen verschiedene jeweils konsistente Interpretationen vor, insofern ist dieses Problem aus instrumentalistischer Sicht gelöst. Das Verschwinden der Vielen Welten muss man im Kontext einer ontischen Sichtweise jedoch erklären - das ist bis heute ungelöst - oder alternativ dazu ihre Existenz akzeptierten.

“Und doch, was bringt’s? Es muß ja mehr als “nur” eine intellektuelle Herausforderung sein, wenn sich hochangesehene Leute damit beschäftigen, oder?
Warum? Ist eine umfassende und konsistente Beschreibung der Vorgänge einschließlich des Messprozesses nicht genug Ansporn?

Timm
22.06.21, 10:37
Warum? Ist eine umfassende und konsistente Beschreibung der Vorgänge einschließlich des Messprozesses nicht genug Ansporn?
Siehst du Anzeichen, dass man den Messprozess jemals verstehen wird oder die Existenz der VW jemals verifizieren kann?

Birgt die Bezeichnung "Interpretationen" nicht schon die vorausschauende Erkenntnis der Nicht-Beweisbarkeit in sich?

Nun könntest du mit dem Inneren von Schwarzer Löchern argumentieren und ja, man kann nicht nachschauen ob die t- und r-Koordinaten tatsächlich vertauscht sind. Doch kann man prinzipiell dort hin, was für die VW nicht zutrifft. Und das macht den Unterschied.

TomS
22.06.21, 11:34
Siehst du Anzeichen, dass man den Messprozess jemals verstehen wird oder die Existenz der VW jemals verifizieren kann?

Birgt die Bezeichnung "Interpretationen" nicht schon die vorausschauende Erkenntnis der Nicht-Beweisbarkeit in sich?

Gegenfrage: siehst du Anzeichen, dass man den Messprozess niemals verstehen oder die Existenz der VW explizit falsifizieren kann?

Der Begriff der Interpretationen wurde geprägt, um eben diese „vorausschauende Erkenntnis“ der Nicht-Beweisbarkeit anzudeuten. Warum aber sollte man sich von diesem Vorurteil einengen lassen?

Ich denke, man wird den Messprozess auf Basis der VW mathematisch entweder vollumfänglich verstehen können, oder man wird zeigen können, wo dies fehlschlägt. Im Erfolgsfall wäre die tatsächliche physikalische Existenz der VW natürlich nicht verifiziert, die Hypothese der Existenz wäre dann jedoch mindestens so plausibel wie die der Nichtexistenz. Warum also sollte man vorzeitig aufgeben?

Man muss ja nicht an die VWI als Wahrheit glauben, aber man kann sie doch als Forschungsprogramm ernst nehmen. Warum sollte man nicht in diese Richtung arbeiten, nur weil nicht alle Physiker davon überzeugt sind?

Schau dir mal die Geschichte um die Arbeiten von Bell an. Wenn es nach den damals tonangebenden Köpfen gegangen wäre, hätte er daran niemals arbeiten dürfen.

Timm
22.06.21, 17:29
Gegenfrage: siehst du Anzeichen, dass man den Messprozess niemals verstehen oder die Existenz der VW explizit falsifizieren kann?

Der Begriff der Interpretationen wurde geprägt, um eben diese „vorausschauende Erkenntnis“ der Nicht-Beweisbarkeit anzudeuten. Warum aber sollte man sich von diesem Vorurteil einengen lassen?
Solange Versuche scheitern die Quantenmechanik zu erweitern, verstehe ich die erste Frage als rein rhetorisch. Und solange sollte man denke ich nicht von einem Vorurteil sprechen.

Ich denke, man wird den Messprozess auf Basis der VW mathematisch entweder vollumfänglich verstehen können, oder man wird zeigen können, wo dies fehlschlägt. Im Erfolgsfall wäre die tatsächliche physikalische Existenz der VW natürlich nicht verifiziert, die Hypothese der Existenz wäre dann jedoch mindestens so plausibel wie die der Nichtexistenz. Warum also sollte man vorzeitig aufgeben?
Ein "vollumfängliches" Verständnis des Messprozesses könnte genauso gut die VW ad absurdum führen. Aber die grundlegendere Frage ist doch, ob es da überhaupt etwas zu verstehen gibt. Verstehen wir die Nicht-Lokalität? Werden wir sie jemals verstehen?
In unserem Bemühen alles verstehen zu wollen, bürden wir uns sonderbare Krücken auf wie die, einer Rechenvorschrift ontische Eigenschaften zuzubilligen. Etwas, das mich zunehmend am meisten stört.

Man muss ja nicht an die VWI als Wahrheit glauben, aber man kann sie doch als Forschungsprogramm ernst nehmen. Warum sollte man nicht in diese Richtung arbeiten, nur weil nicht alle Physiker davon überzeugt sind?

Ich würde die VWI als Forschungsprogramm ernster nehmen, wenn es Fortschritte gäbe, die darauf hin deuten, dass es die VW tatsächlich gibt und das "I" somit entfallen kann.
Solange das nicht der Fall ist, scheint mir hier eine Verschwendung von Talenten stattzufinden, die anderswo dringend benötigt werden. Wie die Neutrino Forschung zeigt, ist das SM nicht in trockenen Tüchern und es gibt mehr, du weißt das besser.

TomS
22.06.21, 19:28
Solange Versuche scheitern die Quantenmechanik zu erweitern ...
Ich sprach nie davon, die Quantenmechanik zu erweitern. Meinst du z.B. GRW?

Ein "vollumfängliches" Verständnis des Messprozesses könnte genauso gut die VW ad absurdum führen.
Nicht gleich "ad absurdum führen"; Gültigkeitsgrenzen aufzeigen, falsifizieren, ...

Aber die grundlegendere Frage ist doch, ob es da überhaupt etwas zu verstehen gibt ...
In unserem Bemühen alles verstehen zu wollen, bürden wir uns sonderbare Krücken auf wie die, einer Rechenvorschrift ontische Eigenschaften zuzubilligen. Etwas, das mich zunehmend am meisten stört.
Ich weiß, dass dich das stört, aber ich verstehe das Problem nicht.

Es geht einfach nur darum, die Quantenmechanik so zu verstehen wie zum Beispiel die Gravitation oder den Elektromagnetismus. Da stört sich auch niemand daran, dass man die Selbstenergie des Elektrons verstehen möchte und dass dies mittels der Maxwellschen Theorie nicht sowie mittels der Quantenelektrodynamik nur eingeschränkt funktioniert. Niemand stellt das Dogma auf, man dürfe oder könne die Selbstenergie des Elektrons nicht verstehen; man erprobt verschiedene Ansätze, evaluiert deren Gültigkeitsbereich. So funktioniert Physik.

Es hat sich auch noch niemand daran gestört, wenn man denn Elektromagnetismus ontisch auffassen möchte. Erst mit der Quantenmechanik - ca. zeitgleich mit dem einflussreichen logischen Positivismus - wurde es schick, bestimmte Denkansätze verbieten zu wollen.

Wohlan, man muss sich an diesem Nachdenken nicht beteiligen, wenn man nicht möchte, aber man sollte akzeptieren, dass es eben Physiker gibt, die dies sehr wohl als relevant erachten (Einstein, Bohm, Bell, Penrose ...).

Ich würde die VWI als Forschungsprogramm ernster nehmen, wenn es Fortschritte gäbe, die darauf hin deuten, dass es die VW tatsächlich gibt.
Die gibt es - mit jedem Phänomen, das mittels Selbstinterferenz eines Quantensystems innerhalb einer räumlich ausgedehnten Apparatur erklärt werden kann. Bei elektromagnetischen Wellen hast du damit evtl. kein Problem, obwohl man die auch nie direkt sieht, immer nur vermittelt durch die Verarbeitung einzelner elektrochemischer Signale im Gehirn. Bei Photonen kommst du ins Grübeln, bei Katzen magst du nicht mehr.

Entweder erklärst du, warum du hier eine Unterscheidung machst, oder du lehnst eine ontische Interpretation prinzipiell ab. Letzteres ist eine valide philosophische Postion, aber keine alternativlose.

Solange das nicht der Fall ist, scheint mir hier eine Verschwendung von Talenten stattzufinden, die anderswo dringend benötigt werden. Wie die Neutrino Forschung zeigt, ist das SM nicht in trockenen Tüchern und es gibt mehr, du weißt das besser.
Das halte ich für eine ganz gefährliche Argumentation.

Wie gesagt, lies mal etwas über die Ansicht der Kopenhagener noch in den fünfziger Jahren insbs. zu den Bellschen Arbeiten; einige hielten dies auch für Verschwendung.

Ähnliches gilt für die Arbeiten von Zeh zur Dekohärenz, die heute als etabliert gilt. Sämtliche neuen mathematischen Erkenntnisse bleiben unabhängig von der jeweils angewandten Interpretation bestehen.

Wer entscheidet eigentlich, für welches Problem die paar Dutzend Physiker, die sich mit der Viele-Welten-Interpretation befassen, besser zu verwenden wären? Warum nimmt man nicht lieber einige tausend Stringtheoretiker?

Quantor
22.06.21, 19:42
Die Interpretationen der QM erzählen Gegensätzliches über das Zustandekommen von Messresultaten.

Einigkeit besteht darin, daß die Messresultate den Vorhersagen der QM entsprechen.

Welchen wissenschaftlichen Fortschritt erhofft man sich von Forschung über Interpretationen der QM?

Die Interpretationen sagen physikalisch gar nichts aus, das ist reine Philosophie.
Warum man definite Messwerte nach quantenmechanischen Wahrscheinlichkeiten misst und nicht Überlagerungszustände, wie sie durch den (unitären) Formalismus beschrieben werden, kann so überhaupt nicht erklärt werden.
Es gibt Ansätze, diese "klassischen" Messzustände zu erklären, zB. durch Dekohärenz, aber das ist völlig unabhängig von der Interpretation.
Es gibt euch eine Formulierung der Quantenmechanik, die Wentzel-Feynmannsche Pfadintegralmethode, die über alle Wahrscheinlichkeitspfade integriert, so dass mit der gesamten Information der Wellenfunktion letztlich genau ein solcher "klassischer" Zustand unter experimentellen Bedingungen resultiert, analog dem Huygens-Fresnelschen Prinzip der Elektrodynamik. Allerdings muss man auch hier Annahmen über den klassischen Zustand machen.
Letztlich sind solche Interpretationen vielleicht naturphilosophisch bedeutsam, um heuristische Annahmen über eine übergreifende Theorie zu machen, die dann über die Quantenmechanik hinausgeht.

TomS
22.06.21, 19:54
Die Interpretationen sagen physikalisch gar nichts aus, das ist reine Philosophie.
Das ist eine Frage der Auffassung des jeweiligen Physikers. Einstein betrachtet seinen realistischen Anspruch an die Physik durchaus als physikalische Fragestellung, Bohr et al. dagegen nicht.

Zu allem weiteren absolute Zustimmung.

Quantor
22.06.21, 20:02
Das ist eine Frage der Auffassung des jeweiligen Physikers. Einstein betrachtet seinen realistischen Anspruch an die Physik durchaus als physikalische Fragestellung, Bohr et al. dagegen nicht.


Einstein war gefangen in seinen Vorstellungen einer einheitlichen geometrischen Feldtheorie, mit der er die "Quantenprobleme" zu erschlagen hoffte. Man kann das Konzept aber als gescheitert betrachten. Es war aber durch eine "realistische" Naturbeschreibung motiviert, die gleichzeitig auch für ihn Lokalität bedeutete.

Bohr war da sicher "positivistischer" orientiert, allerding wohl nicht aus Prinzip sondern aus Notwendigkeit physikalischer Fakten.

Und letztlich, im Tatbestand der Fakten muss man sogar der Bohrschen Schule (derzeit noch) recht geben.

Quantor
22.06.21, 20:21
Also ich meine, dass die "Interpretationsfragen" zur Quantenmechanik schon wichtig sind, aber nicht, um die QM zu verbessern, sondern eher, um (mögliche) Erweiterungen besser zu verstehen.
So ist m.E. weiterhin an "realistischen" Theorien festzuhalten, aber die Nichtlokalität werden wir wohl nicht mehr los werden, damit werden auch zukünftige Theorien leben müssen.
Wie immer wird die "Wahrheit" irgendwo in der Mitte liegen, zwischen Einstein und Bohr..

TomS
22.06.21, 20:37
Einstein war gefangen in seinen Vorstellungen einer einheitlichen geometrischen Feldtheorie …
Später, aber noch nicht zu Zeiten der Solvay-Konferenzen.

Er war aber durch eine "realistische" Naturbeschreibung motiviert, die gleichzeitig auch für ihn Lokalität bedeutete.
Ersteres ja, letzteres kann ich so nicht sagen.

Bohr war da sicher "positivistischer" orientiert, allerding wohl nicht aus Prinzip sondern aus Notwendigkeit physikalischer Fakten.
Wieso der Notwendigkeit physikalischer Fakten? Diese lassen sicher auch andere Positionen zu. Ich denke eher, eine Mischung aus Grundhaltung und Pragmatismus. Der physikalische Fortschritt wäre nicht innerhalb weniger Jahre so enorm gewesen, wenn man nicht bestimmte Fragestellungen praktisch vollständig ausgeblendet hätte.

Und letztlich, im Tatbestand der Fakten muss man sogar der Bohrschen Schule (derzeit noch) recht geben.
Keineswegs.

Das erste konkrete Ergebnis zu Berechnungen ohne die Notwendigkeit eines Kollapses stammt von Mott aus 1929:

https://en.m.wikipedia.org/wiki/Mott_problem
https://arxiv.org/abs/1209.2665

Everett zeigte bekanntermaßen in den Fünfzigern eine völlig äquivalente Alternative zur orthodoxen Interpretation auf.

Gleasons Theorem (1957) und der Hartle-Finkelstein Frequency Operator (1960iger) legen nahe, dass das Postulat der Bornschen Regel nicht fundamental ist:

https://en.m.wikipedia.org/wiki/Gleason%27s_theorem
https://arxiv.org/abs/1105.4499

Die Bohrschen Schule hat angesichts der Dekohärenz (Zeh, Ende der 60iger, später Zurek) durchaus Probleme, das Kollapspostulat vernünftig aufrechtzuerhalten. Es gibt alternative Weiterentwicklungen, z.B. die Consistent histories.

Von der Bohrschen Schule im engeren Sinne ist nicht mehr viel übrig, im weiteren Sinne einer eher instrumentalistischen Position natürlich schon.

Quantor
22.06.21, 20:55
Von der Bohrschen Schule im engeren Sinne ist nicht mehr viel übrig, im weiteren Sinne einer eher instrumentalistischen Position natürlich schon.

Es kommt wohl darauf an, was man da unter "Bohrscher Schule" versteht. Eigentlich sehe ich das nicht ganz so homogen, wie es immer adressiert wird.
Sicher hat von Neumann mit seinem Werk da einen gewissen Standard geschaffen. Im Zusammenhang mit seinem Aufenthalt bei Hilbert hat er da eine gewisse Axiomatik formuliert, die heutzutage so nicht mehr allgemein vertreten wird, dazu gehören Projektionen auf Eigenzustände im Zusammenhang mit Messungen, was ja letztlich den "Kollaps" charakterisiert.
Unter der "Bohrschen Schule" verstehe ich aber erstmal die Ablehnung, "klassische Konzepte" wie zB. Teilchenbahnen auf quantenmechanische Systeme anwenden zu können, dann natürlich auch die Schlagwörter von "Komplementarität", "Dualismus" und "Korrespondenz".
Letztlich bedeutet "Bohsche Schule" für mich, dass die "Quantenrealität" eben keine "klassische Realität" sein kann, der klassische Zustandsbegriff ist da nutzlos. Andererseits gehört für mich zur "Bohrschen Schule" auch die Existenz klassischer Systeme anzuerkennen, die immer im Zusammenhang mit Messungen definite Messwerte festlegt. Das verbindende Glied ist da die "Bornsche Regel", die aus dieser "Quantenrealität" eine "klassische Realität" liefert, aber eben nur nach Wahrscheinlichkeiten. Was dann als "Quantenrealität" im (positiven) Sinne der "Bohrschen Schule" betrachtet wird, scheint mir nicht so allgemein bestimmt, das kommt auf den Vertreter dieser Schule an.

TomS
22.06.21, 21:28
Das gehe ich im wesentlichen mit.

Mir war nur wichtig, genau auf diese Heterogenität, Brüche und Weiterentwicklungen hinzuweisen. Leider wird von der sogenannten orthodoxen Interpretation zu oft eine grobe Skizze gezeichnet und dies dann als die endgültige Wahrheit über die Quantenmechanik dargestellt.

Timm
22.06.21, 21:40
Ich weiß, dass dich das stört, aber ich verstehe das Problem nicht.

Es geht einfach nur darum, die Quantenmechanik so zu verstehen wie zum Beispiel die Gravitation oder den Elektromagnetismus. Da stört sich auch niemand daran, dass man die Selbstenergie des Elektrons verstehen möchte ...
Bei der Selbstenergie des Elektrons geht es im Ergebnis um Ladungsverteilung. Willst du sagen, wenn man die Ladungsverteilung eines Elektrons ontisch versteht (ein gutes Beispiel), dann sollte man die Wellenfunktion eines Elektrons, mit der man Wahrscheinlichkeiten ausrechnet ebenfalls ontisch verstehen? Dem kann ich nicht folgen.


Die gibt es - mit jedem Phänomen, das mittels Selbstinterferenz eines Quantensystems innerhalb einer räumlich ausgedehnten Apparatur erklärt werden kann....
"Die gibt es" bezieht sich auf "Fortschritte, die darauf hin deuten, dass es die VW tatsächlich gibt."
Was hat die Selbstinterferenz (Beispiel Doppelspalt) mit den akausal getrennten VW zu tun?



Entweder erklärst du, warum du hier eine Unterscheidung machst, oder du lehnst eine ontische Interpretation prinzipiell ab. Letzteres ist eine valide philosophische Postion, aber keine alternativlose.
Ich lehne eine ein ontische Interpretation nicht prinzipiell ab, s. oben. Aber weshalb sollte eine Rechenvorschrift, die die Wahrscheinlichkeit liefert, ein Quantenobjekt an einem bestimmten Ort zu finden, ontisch zu verstehen sein.

Wer entscheidet eigentlich, für welches Problem die paar Dutzend Physiker, die sich mit der Viele-Welten-Interpretation befassen, besser zu verwenden wären? Warum nimmt man nicht lieber einige tausend Stringtheoretiker?
Das wäre mir auch recht.

Dass es nur ein paar Dutzend sind, wusste ich nicht. Verfolgst du deren Arbeiten? Denen sei ihr Hobby gegönnt. ;)

Quantor
22.06.21, 21:46
Mir war nur wichtig, genau auf diese Heterogenität, Brüche und Weiterentwicklungen hinzuweisen. Leider wird von der sogenannten orthodoxen Interpretation zu oft eine grobe Skizze gezeichnet und dies dann als die endgültige Wahrheit über die Quantenmechanik dargestellt.

Ja, die damalige Frage im Zusammhang mit der "Bohrschen Schule" war ja gerade, ob die Quantenmechanik die Natur "vollständig" beschreibt. Genau das hat ja Einstein angezweifelt. Er hat also nicht die Quantenmechanik und ihre Interpretation durch Bohr an sich angezweifelt, sondern, dass sie die "Quantenrealität" vollkommen beschreibt. Einstein war der Auffassung, dass es eine umfassendere Theorie der "Quantenrealität" geben muss, welche die (nach ihm) Unzulänglichkeiten dieser Theorie beseitigt, also Realität, Lokalität und Nicht-Indeterminsimus wieder herstellt.
Alle anderen heutigen Interpretationen der Quantenmechanik sind da aber im Sinne Bohrs, auch wenn sie dessen Interpretation der Quantenmechanik modifizieren. Ich denke, die "Bohrsche Schule" sollte man auch aus diesem historischen Kontext aus betrachten.

TomS
22.06.21, 22:59
Ich lehne eine ein ontische Interpretation nicht prinzipiell ab, s. oben. Aber weshalb sollte eine Rechenvorschrift, die die Wahrscheinlichkeit liefert, ein Quantenobjekt an einem bestimmten Ort zu finden, ontisch zu verstehen sein.
Woraus folgt denn logisch zwingend, dass die Quantenmechanik ausschließlich stochastisch verstanden werden kann? Weshalb sollte eine Rechenvorschrift, die eine Wahrscheinlichkeit liefert, nicht fundamental ontisch interpretiert werden können? Die Hamiltonsche Mechanik mit der Phasenraumdichte und der Liouville-Gleichung zeigt, dass beides möglich ist; hier behauptet niemand, dieser Formalismus müsse zwingend stochastisch interpretiert werden. Dass der Formalismus der Quantenmechanik stochastisch interpretiert werden kann - OK. Dass er zwingend so interpretiert werden muss - ein Vorurteil.

Es geht darum, eine physikalische Fragestellung zu klären. Wie erklärt die QED die Selbstenergie des Elektrons? Wie erklärt die Quantenmechanik den Messprozess? Ich sehe keinen prinzipielle Unterschied.

In beiden Fällen geht es darum, den vorliegenden Formalismus anzuwenden, um die jeweiligen Prozesse präzise zu beschreiben. Dabei stößt man selbstverständlich an gewisse Grenzen, die auf Probleme im jeweiligen Formalismus, dessen Unvollständigkeit oder gar Inkonsistenz hinweisen. in beiden Fällen sind wir uns einig, dass die Probleme heute nicht vollumfänglich verstanden sind. Während man im Falle des Selbstenergie davon ausgehen kann, dass die QED aufgrund mathematischer Unzulänglichkeiten tatsächlich nicht ausreichend ist, existiert im Rahmen der Quantenmechanik kein prinzipieller Grund oder gar ein Beweis, die Unmöglichkeit des Verständnisses der Messung zu behaupten.

Nun verfestigt sich jedoch seit Mitte der zwanziger Jahre die Meinung, dieses Problem sei prinzipiell unlösbar, man benötige zwingend eine durch den quantenmechanischen Formalismus nicht beschreibbare klassische Welt, die Fragestellung sei grundsätzlich unzulässig oder unphysikalisch usw.

Betrachtet man die spätere Entwicklung, insbs. die Dekohärenz, so zeigt sich jedoch, dass ein Teil genau der Fragestellungen explizit gelöst werden, die ein halbes Jahrhundert zuvor als unlösbar oder unzulässig deklariert wurden.

Offenbar war in den zwanziger Jahren nicht absehbar, wie leistungsfähig der Formalismus tatsächlich ist; das ist nicht schlimm. Schlimm ist jedoch, neue Erkenntnisse nicht zur Kenntnis zu nehmen.

Born interpretierte die Wellenfunktion wahrscheinlichkeitstheoretisch; dass dies zulässig ist, bedeutet nicht logisch zwingend, dass dies die einzige zulässige Interpretation ist. Dass eine Rechenvorschrift eine Wahrscheinlichkeit liefert, bedeutet nicht logisch zwingend, dass nicht auch deterministische Deutungen möglich sind. Niemand behauptet ernsthaft, die Hamiltonsche Mechanik sei zwingend rein stochastisch, nur weil eine entsprechende Formulierung möglich ist.

Born postuliert ein spezielles Wahrscheinlichkeitsmaß; Gleason beweist jedoch, dass das Postulat in dieser Form unnötig ist. Was würdest zu einem Axiom sagen, das eine eindeutige Zerlegung der natürlichen Zahlen in Primfaktoren postuliert, wenn du später lernst, dass genau dies ein Theorem in einem schwächeren Axiomensystem darstellt? Ob du dies als Axiom oder Theorem betrachtest, spielt für die praktische Anwendung keine Rolle, da das Axiom mit dem Theorem verträglich bzw. sogar identisch. Dennoch würde niemand ignorieren, wenn ein Axiom überflüssig ist oder vereinfacht werden kann.

Du betrachtest alles durch die Brille der orthodoxen Interpretation nach Bohr, von Neumann et al., und dir entgeht dabei, dass nichts davon logisch zwingend ist, dass Alternativen existieren - auch wenn Bohr dies vehement bestritten hat - dass einiges explizit überholt ist, dass ggf. schwächere Postulate möglich sind.

Natürlich darfst du diese Sichtweise einnehmen, sie ist zulässig und man kann daraus eine zutreffende Interpretation basteln. Man darf jedoch nicht glauben, diese Sichtweise sei irgendwie alternativlos, logischer, stringenter, einfacher oder sonst irgendwas. Es ist einfach eine lose Ansammlung teilweise unpräziser Glaubenssätze, von denen keiner in Stein gemeißelt ist. Seither sind fast 100 Jahre vergangen, unser Wissen um die Fundamente der Quantenmechanik hat sich seither enorm entwickelt.

Was hat die Selbstinterferenz (Beispiel Doppelspalt) mit den akausal getrennten VW zu tun?
Was meinst du mit akausal?

Die Selbstinterferenz hat enorm viel mit den Vielen Welten zu tun. Ob eine Superposition eines Photons entlang zweier Lichtwege oder einer Katze vorliegt, ist nur ein gradueller Unterschied, kein prinzipieller.

Wenn du H2O-Moleküle betrachtest, dann entspricht dies der fundamentalen Beschreibung; Wasserwellen, Dampf oder Eiskristalle sind daraus resultierende makroskopische Phänomene. Genauso verhält es sich gemäß der VWI: die Realität wird durch einen Quantenzustand vollständig und zutreffend beschrieben; es existieren verschiedene Ausprägungen, z.B. Katzen, jedoch kein fundamentaler Unterschied.

An sich ist diese Aussage der VWI so unendlich viel einfacher als alle anderen Aussagen von Bohr et al.

TomS
22.06.21, 23:05
[Einstein] hat also nicht die Quantenmechanik und ihre Interpretation durch Bohr an sich angezweifelt, sondern, dass sie die "Quantenrealität" vollkommen beschreibt. Einstein war der Auffassung, dass es eine umfassendere Theorie der "Quantenrealität" geben muss, welche die (nach ihm) Unzulänglichkeiten dieser Theorie beseitigt, also Realität, Lokalität und Nicht-Indeterminsimus wieder herstellt.
Alle anderen heutigen Interpretationen der Quantenmechanik sind da aber im Sinne Bohrs, auch wenn sie dessen Interpretation der Quantenmechanik modifizieren.
Mit Ausnahme der (modernen Ausprägung der) Everettschen Quantenmechanik.

Sie stellt insofern eine interessante Alternative dar, als sie eine realistische und in gewisser Weise - nicht im Einsteinschen Sinne - vollständige Interpretation liefert, dabei jedoch keine umfassendere Theorie erfordert.

Timm
23.06.21, 19:58
Dass der Formalismus der Quantenmechanik stochastisch interpretiert werden kann - OK. Dass er zwingend so interpretiert werden muss - ein Vorurteil.
Wie müssen uns davon frei machen, im Rahmen von Interpretationen von "zwingend" oder von einem "Vorurteil" zu sprechen.

Es geht darum, eine physikalische Fragestellung zu klären. Wie erklärt die QED die Selbstenergie des Elektrons? Wie erklärt die Quantenmechanik den Messprozess? Ich sehe keinen prinzipielle Unterschied.
Wir vergleichen die Selbstenergie mit der Wellenfunktion (nicht mit dem Messprozess), wenn es um die "ontische" Frage geht, s.o.

Betrachtet man die spätere Entwicklung, insbs. die Dekohärenz, so zeigt sich jedoch, dass ein Teil genau der Fragestellungen explizit gelöst werden, die ein halbes Jahrhundert zuvor als unlösbar oder unzulässig deklariert wurden.
Stützt die Dekohärenz eine der Interpretationen hinsichtlich Beweisbarkeit? Ich sehe das nicht. Das war aber der Ausgangspunkt. Die Dekohärenz trägt auch nichts zum Verständnis des Messprozesses bei.

Natürlich darfst du diese Sichtweise einnehmen, sie ist zulässig und man kann daraus eine zutreffende Interpretation basteln. Man darf jedoch nicht glauben, diese Sichtweise sei irgendwie alternativlos, logischer, stringenter, einfacher oder sonst irgendwas.
Ich bevorzuge die instrumentelle Interpretation, halte sie jedoch keineswegs für zwingend.

Aus dem Bisherigen sehe ich nicht, dass es Fortschritte gibt, die zur Beweisbarkeit einer der Interpretation führen könnten. Aus deinen Ausführungen erkenne ich das zumindest nicht.

TomS
23.06.21, 20:53
Natürlich trägt die Dekohärenz maßgeblich zum Verständnis des Messprozesses bei; sie erklärt mittels der Schrödingergleichung, welche makroskopisch zulässigen Zeiger-Zustände entstehen können.

Beweisbarkeit kannst du von keiner physikalischen Theorie erwarten, lediglich von ihrem mathematischen Kern.

Es geht um Plausibilität; das war schon bei Newton vs. Kepler (Epizyklen) der Fall.

Timm
24.06.21, 08:55
Natürlich trägt die Dekohärenz maßgeblich zum Verständnis des Messprozesses bei; sie erklärt mittels der Schrödingergleichung, welche makroskopisch zulässigen Zeiger-Zustände entstehen können.

Aber was trägt sie zum Verständnis der Reduktion des quantenmechanischen Zustands bei? Darum geht es doch beim Messproblem.


Hier (https://physics.stackexchange.com/questions/295527/decoherence-does-it-solve-the-measurement-problem-is-it-discontinuous-when-do) wird Schlosshauer zitiert:

We have argued that, within the standard interpretation of quantum mechanics, decoherence cannot solve the problem of definite outcomes in quantum measurement

Nach meiner Auffassung erklärt das Dekohärenzkonzept das Zustandekommen quasi-klassischen Verhaltens. Ich sehe nicht, dass Dekohärenz eine der Interpretationen der QM stützt. Denn diese scheiden sich am Verständnis des Kollapses. Wenn du das anders siehst, würde ich dich um Referenzen bitten.

TomS
24.06.21, 12:58
Aber was trägt sie zum Verständnis der Reduktion des quantenmechanischen Zustands bei?
Sie liefert Argumente, warum dieser ontisch verzichtbar ist, zugleich jedoch epistemisch gültig bleibt. Siehe unten.

Darum geht es doch beim Messproblem.
Nur um den Kollaps? Nein, das ist zu eng gefasst.



Zum einen umfasst das Messproblem mehrere Teilprobleme:
1) welche Messergebnisse können auftreten?
2) wann, warum, mit welcher Wahrscheinlichkeit ... tritt ein konkretes Messergebnis auf?
3) wie wird das System während und nach der Messung beschrieben?

Anders gesprochen, zu begründen sind die Postulate
1) Eigenwerte a als mögliche Messwerte
2) Bornsche Regel p(a) = <ψ|a><a|ψ>
3) von Neumannschen Projektionspostulat d.h. Kollaps |ψ> → |a><a|ψ>

Die Dekohärenz besagt:
1) der spezifische Hamiltonoperator des Messgerätes selektiert die möglichen Zeiger-Zustände und Messwerte
2) die Norm je Komponente (Unterraum) liefert die Wahrscheinlichkeit
3) das Projektionspostulat wird zunächst weder postuliert oder abgeleitet noch ausgeschlossen

Damit löst die Dekohärenz den Punkt (1). Sie liefert zusammen mit Gleason's Theorem die eindeutige und phänomenologisch zutreffende mathematische Regel zur Berechnung der Wahrscheinlichkeiten in (2) - bzw. eine geeignete Erweiterung - jedoch keine Begründung, warum überhaupt Wahrscheinlichkeiten auftreten. Außerdem lässt sie einerseits das Kollapspostulat zu, andererseits erlaubt sie auch den Verzicht darauf; im ersten Fall liefert sie wie bei (2) die Unterräume, auf die zu projizieren ist und motiviert zusammen mit (1) die Form von (3); im zweiten Fall löst sie nach Meinung von Zeh, Carroll, Wallace et al. das Problem, warum man von einem epistemischen d.h. lediglich wahrgenommenen Kollaps ausgehen kann, obwohl tatsächlich kein ontischer Kollaps stattfindet.

Die Dekohärenz trägt demnach mittels des selben Formalismus in unterschiedlicher Weise zu verschiedenen Interpretationen bei. Es ist eine Frage der Interpretation, welchen Beitrag wir als natürlicher oder kohärenter empfinden. In den meisten Fällen löst die Dekohärenz jedoch einen Teil der Probleme - Ausnahme s.u.

Insofern Zustimmung zu diesem Statement

We have argued that, within the standard interpretation of quantum mechanics, decoherence cannot solve the problem of definite outcomes in quantum measurement.

bzgl. eines Teilproblems im Rahmen einer speziellen Interpretation. Auch Zeh betont mehrfach, dass die Dekohärenz das Messproblem nicht vollständig löst, in Teilen jedoch zu einer Lösung maßgebliche Beiträge liefert.

(soweit ich mich erinnere bleibt Schlossauer bzgl. der Interpretationen neutral)

Eine mögliche Lesart ist also, dass die Dekohärenz zur Erklärung beiträgt, warum der Kollaps verzichtbar ist. Das umfasst zwei Aspekte: der "ontische" Kollaps ist sicher verzichtbar - siehe unten; ein lediglich "epistemischer" Kollaps wird zumindest teilweise erklärt (Zeh, Carroll, Wallace et al.).



Ich sehe nicht, dass Dekohärenz eine der Interpretationen der QM stützt. Denn diese scheiden sich am Verständnis des Kollapses.
Zunächst mal musst du verstehen, dass der Kollaps im Sinne von Neumanns nach Meinung vieler Physiker verzichtbar ist - über die MWI hinaus. Wenn man ausschließlich an der Berechnung von Wahrscheinlichkeiten interessiert ist, dann sind (1) und (2) ausreichend, (3) ist überflüssig.

Ein explizites Beispiel ist das Mott-Problem, d.h. die Entstehung der Tröpfchenspuren in einer Blasenkammer:

https://en.wikipedia.org/wiki/Mott_problem

Wenn man die Wechselwirkungen des Elementarteilchens mit den Atomen und deren Ionisation als wiederholte Ortsmessung auffasst, dann fordert von Neumann einen Kollaps in einen Ortseigenzustand. Dieser ist bzgl. des Impulses isotrop, d.h. als Ergebnis folgt keine geradlinige Spur sondern ein isotroper Random Walk. In der o.g. Formulierung nach von Neumanns ist (1) demnach explizit falsch.

Mott (1929!) löst das Problem explizit ohne Kollaps. Er berechnet Korrelationen, d.h. letztlich bedingte Wahrscheinlichkeiten P(z | x, y), d.h. die Wsk. für z unter den Annahme x und y. Im Ergebnis ist die Wahrscheinlichkeit für mehrere nicht auf einer Linie liegende Tröpfchen sehr nahe Null.

Heisenberg hat wohl in Richtung von "weak measurements" gedacht und die o.g. Kollapsregel entsprechend modifiziert (ich habe keinen Zugriff auf das Paper). Es sollte darauf hinauslaufen, dass lediglich auf einen Unterraum statt auf einen Ortseigenzustand projiziert wird, dieser Unterraum bzgl. des Impulses anisotrop bleibt und somit eine geradlinige Spur resultiert. Dies hat den Schönheitsfehler, dass die Vorhersagbarkeit leidet, da man unter Verwendung geeigneter Unterräume lediglich ex post eine Erklärung erhält, während Mott ohne weitere Annahmen vorgeht.

Anyway, das Mott-Problem ist ein Beispiel, das üblicherweise als Argument für den Kollaps im Falle aufeinanderfolgender Messungen am selben System angeführt wird, das jedoch in der Tat gegen einen Kollaps verwendet werden kann. Die Argumentation von Mott kommt natürlich ohne Dekohärenz aus.

Neben der MWI kommen insbs. die Consistent Histories sowie die Ensemble Interpretationen ohne Kollaps aus. Ballantine hat letztere maßgeblich entwickelt und zeigt anhand weiterer Beispiele, wie u.a. auch der Quanten-Zeno-Effekt gegen den Kollaps interpretiert werden kann (wiederum habe ich keinen Zugriff auf das Paper).



Wenn man also akzeptiert, dass der Kollaps als mathematische Regel nicht notwendig ist, um korrekte Messwerte und Wahrscheinlichkeiten zu berechnen, dann bleibt darüberhinaus die Frage, ob der Kollaps im ontischen Sinne ebenfalls verzichtbar ist (er wäre dann jedoch logisch inkonsistent).

Dabei lautet die Argumentation der VWI, ausgehend von der Dekohärenz, dass die durch die environment-induced superselction (einselection, Zurek) ausgezeichneten Zweige genau die Eigenschaften haben, die man für einen "epistemischen" Kollaps benötigt. Im Falle der Nebelkammer würde die Dekohärenz die in der Projektion zu verwendende Unterräume liefern.

D.h. ich kann entweder
a) die Dekohärenz nutzen, um die Unterräume zu berechnen, auf die ich im Zuge eines Kollapses projiziere, oder
b) nicht projizieren
In beiden Fällen erhalte ich die selben Beobachtungen.

(betrachte zwei Wellenpakete - klassisches Licht - die sich in entgegengesetzter Richtung auseinander bewegen; ob du als Beobachter eines Wellenpaketes das andere wegpostulierst oder seine Existenz akzeptierst läuft bzgl. der Phänomenologie auf's selbe raus)

Die Aussage der MWI lautet also im wesentlichen, dass sich für die Quantenmechanik ohne (1) bis (3) sowie unter Einbeziehung der Dekohärenz ein axiomatisch einfacheres und insgs. schlüssigeres Bild ergibt, als für die Quantenmechanik mit (1) und (2); dem kann man zumindest in soweit folgen, als man schlecht Postulate wie (1) oder (2) ad hoc einführen kann, wenn man erst mittels komplizierter Berechnung die Entitäten erhält, die in (1) oder (2) zu verwenden sind. Die Dekohärenz verkompliziert die Argumentation an dieser Stelle maßgeblich - weil sie einen Teil der Probleme löst (klingt komisch, ist aber so).



Einige Dinge sollten klar sein: der Kollaps ist in praktisch jeder Form verzichtbar; er verbleibt lediglich als praktische Rechenregel, deren Anwendbarkeit aber je Einzelfall zu prüfen ist; die Dekohärenz sowie andere Effekte liefern dafür überzeugende Argumente; in einigen Fällen verkompliziert der Kollaps die Argumentation (nach Ballantine wird sie sogar inkonsistent oder falsch); einige nicht-ontische sowie die MWI als ontische Interpretation kommen explizit ohne Kollaps aus; keine Interpretation, die die Dekohärenz ignoriert, liefert einen Beitrag zu einer echten Erklärung des Messproblems; jeder Versuch einer Kollapsinterpretation erscheint heute doch sehr fragwürdig; jeder Erklärungsversuch, der die hier diskutieren Punkte nicht berücksichtigt, kann nicht ernstgenommen werden.

TomS
24.06.21, 16:05
Hier ein kurzer Artikel zu Mott‘s Arbeit:

https://arxiv.org/abs/1209.2665
Emergence of classical trajectories in quantum systems: the cloud chamber problem in the analysis of Mott (1929)
We analyze the paper "The wave mechanics of α-ray tracks" (Mott, 1929), published in 1929 by N.F. Mott. In particular, we discuss the theoretical context in which the paper appeared and give a detailed account of the approach used by the author and the main result attained. Moreover, we comment on the relevance of the work not only as far as foundations of Quantum Mechanics are concerned but also as the earliest pioneering contribution in decoherence theory.

Mott betrachtet ein abgeschlossenes Quantensystem, das aus einem Alphateilchen sowie den Atomen der Nebelkammer besteht. Er führt keinen Beobachter ein, er verwendet weder Heisenberg-Schnitt zwischen Quantensystem (Alphateilchen) und Messgerät (Nebelkammer) noch einen Kollaps a la von Neumann. Mott modelliert das „Messgerät“ explizit quantenmechanisch, im Gegensatz zu Bohrs Überzeugung, dass dies nicht möglich sei. Letztlich verzichtet er damit auf all die unnötigen Zutaten, die leider über Jahrzehnte als integrale Bestandteile und vermeintliche Rätsel der Quantenmechanik angesehen wurden – und heute noch werden.

Der Artikel ist absolut lesenswert. Er vergleicht den expliziten, quantitativen und recht offensichtlichen Ansatz nach Mott ohne Kollaps mit den teilweise nur qualitativen Argumenten Heisenbergs mit Kollaps. Das Ergebnis – der Kollaps sei in seiner einfachen Form explizit falsch, in einer erweiterten Form weiterhin nicht präzise formuliert und nach der Lösung Mott’s ohnehin überflüssig – ist offensichtlich.

Letztlich kann dieses Modell als sehr einfache Vorstufe zu Modellen für die Dekohärenz angesehen werden.

TomS
24.06.21, 16:24
Hier findest du eine Erklärung auf Basis von „weak measurements“ und einem entsprechenden modifizierten Kollapspostulat. Ob das mathematisch funktioniert wird nie vorgerechnet, die einfache Lösung von Mott leider nicht zur Kenntnis genommen:

https://physics.stackexchange.com/questions/479475/are-bubble-chamber-tracks-inconsistent-with-quantum-mechanics

Timm
24.06.21, 18:08
Nur um den Kollaps? Nein, das ist zu eng gefasst.
Wo ich auch nachschaue, geht es beim Messproblem um die Frage ob und wie der Kollaps der Wellenfunktion stattfindet. Und genau diese Frage löst die Dekohärenz nicht, was deine Aussage "das Projektionspostulat wird zunächst weder postuliert oder abgeleitet noch ausgeschlossen" bestätigt.

Am Kollaps und insbesondere an der Frage ob es ihn gibt oder nicht scheiden sich die Interpretationen, zumindest VWI vs. Kopenhagen. Ich erkenne in deinen Ausführungen nicht, welche Erkenntnisse der letzten Jahrzehnte konkret einer Lösung dieser Frage näher kommen. Darum ging es. Du erwähnst "Meinungen" dieser und jener Experten auf diesem Gebiet. Auch Zeilinger, den ich schon einige male erwähnt habe, hat seine Meinung. Dagegen ist ja nicht das Geringste einzuwenden. Aber wir sind uns sicherlich einig, dass Meinungen die adressierte Frage nicht lösen.

Angesichts deiner investierten Zeit und Mühe, wofür ich mich bedanke, beschleicht mich zunehmend ein schlechtes Gewissen.

TomS
24.06.21, 18:40
Hallo Timm,

ich fürchte, du suchst zu viel und schaust auch an den falschen Stellen nach. Ich bestreite keineswegs, dass das Messproblem häufig um den Kollaps herum aufgezäumt wird; ich bestreite lediglich, dass das sinnvoll ist (und ja, ich behaupte, dass du zu dem Thema sicher viel Quatsch finden wirst).

Anstatt sich mit dem Kollaps rumzuplagen, führt man ihn erst gar nicht ein. Er ist überflüssig bis nutzlos, führt zu vermeidbaren Problemen und ist in der oft zitierten einfachen Fassung beweisbar falsch. Man kommt einer Lösung des Messproblems am besten dadurch näher, dass man von vorneherein auf ihn verzichtet. Viele moderne Interpretationen sind sich diesbzgl. einig.

Wenn man eine ontische Interpretation anstrebt, dann kann man sich mit einem rein epistemischen Kollaps befassen. Da du die MWI nicht magst, zu der das wohl zwangsläufig führt, musst du diesen Schritt aber nicht mitgehen.

Angesichts deiner investierten Zeit und Mühe, wofür ich mich bedanke, beschleicht mich zunehmend ein schlechtes Gewissen.
Kannst du wieder gutmachen :)

Lies dir den oben verlinkten Artikel durch. Mott‘s Rechnung ohne Kollaps ist präzise und klar. Außer der Schrödingergleichung sowie einer minimalen stochastischen Interpretationen mit der Bornschen Regel (2) wird nichts weiter benötigt. Noch einfacher geht’s nicht.

Was bleibt dann deiner Meinung nach für diesen konkreten Fall noch als „Messproblem“ übrig?



Am Kollaps und insbesondere an der Frage ob es ihn gibt oder nicht scheiden sich die Interpretationen, zumindest VWI vs. Kopenhagen.
„Kopenhagen“ ist keine geschlossene Argumentation sondern eine teilweise recht krude und zum Teil überholte Sammlung von Ideen und Dogmen.

Ich erkenne in deinen Ausführungen nicht, welche Erkenntnisse der letzten Jahrzehnte konkret einer Lösung dieser Frage näher kommen.
Komm, ich habe zig Beispiele gebracht.

EDIT: eine Erkenntnis ist, sich besser nicht mit Kopenhagen zu befassen …

Aber wir sind uns sicherlich einig, dass Meinungen die adressierte Frage nicht lösen.
Kannst du deine Frage(n) nochmal präzisieren?

TomS
25.06.21, 06:46
Kannst du deine Frage(n) nochmal präzisieren?
Einen guten Startpunkt bietet m.M.n. das Maudlin-Trilemma.

Maudlin formuliert das Messproblem unabhängig von einem historischen Kontext oder einer speziellen Interpretation; insbs. Kopenhagen spielt bei ihm eine untergeordnete Rolle. Sein unmittelbares Ziel besteht darin, Interpretationen nach ihren Lösungsstrategien für das Messproblem zu kategorisieren.

Maudlin zeigt, dass die folgenden drei Aussagen zusammengenommen inkonsistent sind:
1) Der Zustandsvektor beschreibt das System vollständig
2) Der Zustandsvektor folgt immer einer linearen Zeitentwicklung
3) Messungen haben immer ein definiertes Ergebnis (im Sinne einer definierten Eigenschaft bzgl. einer Observablen)

Theorien mit verborgenen Variablen wie die Bohmsche Mechanik lehnen (1) ab. Die Standard-Quantenmechanik und andere Kollaps-Theorien lehnen (2) ab. Beide führen neben dem Zustandsvektor und der unitären Zeitentwicklung neue Physik ein, erstere verborgenen Variablen, letztere einen Kollaps.

Everett hält an (1 - 2) fest und lehnt (3). Dies führt nicht zu einem einzigen, definierten Ergebnis; stattdessen sind alle quantenmechanisch zulässigen Messergebnisse in je einer Komponente, des Zustandsvektors repräsentiert.

http://www.psiquadrat.de/downloads/maudlin95.pdf

Der Artikel ist lesenswert, insbs. bzgl. der Problemanalyse. Die Bewertung der Lösungsansätze - bereits im Abstract - halte ich dagegen für voreilig: Ansätze wie Consistent Histories sowie die Ensemble-Interpretation kommen zu kurz; bei verborgenen Variablen als auch alternativen Kollaps-Theorien sehe ich kein Potenzial, diese hin zu Quantenfeldtheorien zu verallgemeinern; die knappe Analyse der Everettschen Quantenmechanik führt zwar tatsächlich zum Kern des Problems, Lösungsansätze werden jedoch nicht betrachtet.

Timm
25.06.21, 09:18
ich fürchte, du suchst zu viel und schaust auch an den falschen Stellen nach. Ich bestreite keineswegs, dass das Messproblem häufig um den Kollaps herum aufgezäumt wird; ich bestreite lediglich, dass das sinnvoll ist (und ja, ich behaupte, dass du zu dem Thema sicher viel Quatsch finden wirst).

Anstatt sich mit dem Kollaps rumzuplagen, führt man ihn erst gar nicht ein. Er ist überflüssig bis nutzlos, führt zu vermeidbaren Problemen und ist in der oft zitierten einfachen Fassung beweisbar falsch. Man kommt einer Lösung des Messproblems am besten dadurch näher, dass man von vorneherein auf ihn verzichtet. Viele moderne Interpretationen sind sich diesbzgl. einig.

Wenn man eine ontische Interpretation anstrebt, dann kann man sich mit einem rein epistemischen Kollaps befassen. Da du die MWI nicht magst, zu der das wohl zwangsläufig führt, musst du diesen Schritt aber nicht mitgehen.

Kannst du wieder gutmachen :)

Lies dir den oben verlinkten Artikel durch. Mott‘s Rechnung ohne Kollaps ist präzise und klar. Außer der Schrödingergleichung sowie einer minimalen stochastischen Interpretationen mit der Bornschen Regel (2) wird nichts weiter benötigt. Noch einfacher geht’s nicht.
Ich werde die Rechnung anschauen, ob ich damit klarkomme, ist die andere Frage. Was du hier bzgl. Kollaps ansprichst finde ich hochinteressant. Und deshalb ... .

... ob ich's wieder gutmachen kann, weiß ich nicht. Ich kann dich lediglich einladen, dich an der Diskussion zum Thema Are there signs that any Quantum Interpretation can be proved? (https://www.physicsforums.com/threads/are-there-signs-that-any-quantum-interpretation-can-be-proved.1004469/) zu beteiligen. Du hast offenbar gewichtige Argumente, bringe sie dort ein!

Wenn es generell um die Interpretationen der QM geht, driften Experten schnell auseinander und jeder hat seine Argumente. Bei diesem Thema geht es jedoch nur darum, wie man zweifellos erzielte Fortschritte wertet.

TomS
25.06.21, 11:07
Ich werde die Rechnung anschauen, ob ich damit klarkomme, ist die andere Frage.
Das wichtige an der Rechnung ist lediglich, dass der Kollaps an keiner Stelle eine Rolle spielt, lediglich die Schrödingergleichung sowie implizit die Bornsche Regel.

Was du hier bzgl. Kollaps ansprichst finde ich hochinteressant.
Danke.

Ich kann dich lediglich einladen, dich an der Diskussion zum Thema Are there signs that any Quantum Interpretation can be proved? (https://www.physicsforums.com/threads/are-there-signs-that-any-quantum-interpretation-can-be-proved.1004469/) zu beteiligen.
Das war früher ein Minenfeld und einer der Gründe, warum ich dort raus bin. Es gab diverse Diskussionen, in denen hochangesehen Physiker gezeigt haben, dass sie letztlich völlig ahnungslos sind, keine nicht-technische / nicht-mathematische Diskussion führen können und letztlich immer nur "Kopenhagen X.0" daherfabulieren.

Wenn es generell um die Interpretationen der QM geht, driften Experten schnell auseinander und jeder hat seine Argumente.
Die Frage ist, was denn eigentlich Experten sind. Es gibt viele hochangesehene Experten zur Quantenmechanik, die jedoch keinesfalls Experten zu Interpretationen der Quantenmechanik sind. Und es gibt immer wieder die Tendenz, sich zwar einerseits an diesen Diskussionen zu beteiligen, jedoch andererseits zu betonen, wie unwissenschaftlich, unphysikalisch und sinnlos diese doch sind. Auch die Moderation hat da teilweise sehr parteiisch agiert.

Bei diesem Thema geht es jedoch nur darum, wie man zweifellos erzielte Fortschritte wertet.
Das solltest du jetzt erklären: was genau wären denn deine Kriterien, um diesen Fortschritt zu quantifizieren?

Ich denke nämlich, dass das Hauptproblem darin besteht, diesbzgl. Einigkeit zu erreichen. Evtl. ergibt sich relativ schnell, dass es bzgl. deiner Kriterien keinen Fortschritt gibt, bzgl. meiner schon.

Beispiele:
1) Falsifizierbarkeit im Rahmen konkreten Experimente
2) Verständnis und Erklärungskraft

Timm
25.06.21, 17:34
ie
Das war früher ein Minenfeld und einer der Gründe, warum ich dort raus bin. Es gab diverse Diskussionen, in denen hochangesehen Physiker gezeigt haben, dass sie letztlich völlig ahnungslos sind, keine nicht-technische / nicht-mathematische Diskussion führen können und letztlich immer nur "Kopenhagen X.0" daherfabulieren.
Schade. Es gibt auch solche, die es agnostisch sehen und andere, die eher zur VWI neigen. Aber es ist natürlich deine Sache dort nicht diskutieren zu wollen. Natürlich gibt es unterschiedliche Standpunkte, das macht es ja gerade spannend. Aber wenn hochangesehene Physiker aus deiner subjektiven Sicht so ahnungslos sind ... schwingt da vielleicht etwas persönliches mit.

Ob da viel dabei heraus kommt, wird man sehen. Es geht allenfalls darum, ob Anzeichen erkennbar sind, die die eine oder andere Interpretation plausibler oder unplausibler machen. Wobei ich nicht weiß, ob da wirklich ein objektiver Standpunkt möglich ist.

TomS
25.06.21, 17:53
… schwingt da vielleicht etwas persönliches mit.
Nix schlimmes, es hat mich lediglich genervt.

Wobei ich nicht weiß, ob da wirklich ein objektiver Standpunkt möglich ist.
Nein, da (dort) nicht.

Erstens, weil das prinzipiell nicht geht. Aber auch zweitens, weil die Leute nicht verstehen werden, dass man durchaus unterschiedliche Standpunkte jeweils einzeln objektiv betrachten könnte.

Wir können aber gerne hier zu den beiden Artikeln weiterdiskutieren.

Timm
26.06.21, 09:02
Wir können aber gerne hier zu den beiden Artikeln weiterdiskutieren.
Das Mott Problem habe ich mir flüchtig angeschaut, aber es scheint lohnend, das etwas gründlicher zu machen.

Wikipedia erwähnt " ... illustrating the paradox of the collapse of a spherically symmetric wave function into the linear tracks seen in a cloud chamber."

Ich sehe allerdings zunächst nicht, dass der Kollaps als solcher infrage steht.

P.S. den verlinkten Artikel schaue ich noch an.

Timm
26.06.21, 09:55
Erstens, weil das prinzipiell nicht geht. Aber auch zweitens, weil die Leute nicht verstehen werden, dass man durchaus unterschiedliche Standpunkte jeweils einzeln objektiv betrachten könnte.

Es liegt in der Natur der Sache, dass es bei den Interpretationen unterschiedliche Standpunkte gibt, ja geben muss. Genau das macht die Diskussion dort interessant. Insofern ist unsere "Diskussion" hier etwas einseitig.

Wenn ich nach Anzeichen frage, die die eine oder andere Interpretation stützen, findest solche, die Richtung VWI tendieren.
Anstatt sich mit dem Kollaps rumzuplagen, führt man ihn erst gar nicht ein. Er ist überflüssig bis nutzlos, führt zu vermeidbaren Problemen und ist in der oft zitierten einfachen Fassung beweisbar falsch. Man kommt einer Lösung des Messproblems am besten dadurch näher, dass man von vorneherein auf ihn verzichtet. Viele moderne Interpretationen sind sich diesbzgl. einig.

Völlig natürlich, kein Vorwurf und von mir genau so erwartet. Als ein solches "Anzeichen" wertest du hier die Dekohärenz ("Eine mögliche Lesart ist also, dass die Dekohärenz zur Erklärung beiträgt, warum der Kollaps verzichtbar ist. Das umfasst zwei Aspekte: der "ontische" Kollaps ist sicher verzichtbar - siehe unten; ein lediglich "epistemischer" Kollaps wird zumindest teilweise erklärt (Zeh, Carroll, Wallace et al."-> aus deinem langen Beitrag zur Dekohärenz vom 24.6.), während diese dort so kommentiert wird: " Decoherence has improved the understanding of the measurement problem, but only for the part which recognizes that it does not solve the measurement problem." Wenn sie keinen Beitrag liefert das Messproblem zu lösen. liefert sie nach meiner Meinung auch keinen Beitrag die VWI zu stützen. Andernfalls müsste es Referenzen geben, die genau das thematisieren.

TomS
26.06.21, 10:38
Wikipedia erwähnt " ... illustrating the paradox of the collapse of a spherically symmetric wave function into the linear tracks seen in a cloud chamber."

Ich sehe allerdings zunächst nicht, dass der Kollaps als solcher infrage steht.
Zunächst mal zeigt Mott’s Berechnung explizit, dass hier kein irgendwie gearteter Kollaps benötigt wird, weder zur Berechnung noch zur Interpretation. Er kommt schlicht an keiner Stelle vor.

Historisch betrachtet wurde dieses Experiment gegen den „Kollaps nach Kopenhagen im engeren Sinne“ angeführt. Dieses lautet: Unmittelbar nach einer Messung der Observablen A mit Ergebnis a befindet sich das System in einem Eigenzustand |a> zu A mit eben diesem Eigenwert a.

Dieses Postulat ist im vorliegenden Fall sicher experimentell falsifiziert!

Zunächst mal zur Begründung von Neumanns: Wenn in einer Messung einer Observablen A ein Eigenwert a zu A beobachtet wird und direkt anschließend eine zweite Messung von A durchgeführt wird, so erhält man mit Wahrscheinlichkeit Eins den selben Messwert a. Demnach muss nach der ersten Messung der Eigenzustand |a> vorliegen. Soweit klingt das plausibel.

Das Problem ist zunächst der nach von Neumann zu eng aufgefasste Messprozess, der postuliert, dass immer Eigenwerte zu Observablen auftreten, also exakt scharfe Messwerte und demnach exakt definierte Eigenzustände. Man beachte, dass nach von Neumann die Unschärfe eines makroskopischen Zeigers nicht in die Postulate eingehen.

Wenn wir nun behaupten, dass in der Nebelkammer eine Folge von Ortsmessungen erfolgt, dann erhalten wir nach jeder Messung mit Kollaps (im oben definierten Sinne) sicher Eigenzustände |x1>, |x2> … Diese Ortseigenzustände sind jedoch bzgl. des Impulses isotrop, d.h. in jeder Messung verliert das Quantensystem vollständig die Information über die vorherige „Ausbreitungsrichtung“. D.h. dieser Formalismus bedeutet trivialerweise, dass sich keine geradlinige Tröpfchenspur ausbildet, sondern dass ein Random Walk entsteht - im Widerspruch zur Beobachtung.

Dieser Widerspruch war bereits in den zwanziger Jahren bekannt - siehe das Paper - so dass Heisenberg in Richtung von „weak measurements“ argumentiert. Dabei wird keine exakt scharfe Ortsmessung angenommen, sondern eine „unscharfe“, wobei diese nicht auf die intrinsische quantenmechanische Unschärfe zurückzuführen ist, sondern auf die beschränkte Auflösung des Messgerätes. Dabei erhält man keine scharfen Messwerte, und das Projektionspostulat kann insoweit abgeschwächt werden, als dass nicht mehr auf einen exakten Eigenzustand projiziert wird sondern auf einen (unendlich dimensionalen) Unterraum, so dass letztlich ein bzgl. des Impulses anisotroper Zustand resultiert, der weiterhin eine Richtungsinformation trägt. In der Konsequenz erhält man nach Heisenberg wieder eine geradlinige Tröpfchenspur. Dies wird heute von vielen Physikern, die sich mit dem Messprozess befassen, als die korrekte Beschreibung aufgefasst.

Was also sind die Probleme?

1) Viele Quellen - Skripte zu Vorlesungen etc. - nennen weiterhin ohne Anmerkung die Postulate gemäß „Kopenhagen im engeren Sinne“, betrachten also nur exakt scharfe Messungen. Das ist, wie wir gesehen haben, falsch! Aber deswegen besteht weiterhin dieser Irrglaube auch unter vielen Physiker fort.

2) Die „weak measurements“ lassen sich nicht mehr axiomatisch fassen, denn man kann schlicht nicht mehr mathematisch präzise definieren, auf welchen Unterraum zu projizieren ist. Dies ergäbe sich je System erst mittels expliziter Berechnung, z.B. mittels Dekohärenz. Axiomatisch ist dies ungefähr so unbefriedigend, wie wenn ich im Grundgesetz einen Begriff verwenden würde, der jedoch erst in einer Rechtsverordnung der Stadt Nürnberg definiert ist.

3) Heisenberg argumentiert im Sinne von (2) ohne dies mathematisch präzise auszuführen, folgert daraus jedoch, dass Mott‘s Argumentation gemäß Kopenhagen zulässig sei; offensichtlich wäre es im vorliegenden Fall jedoch genau umgekehrt: wenn Heisenberg seine Methode präzise zu Ende führen würde, dann wäre gezeigt, dass „Kopenhagen“ mit der präzisen Berechnung Mott‘s und dem Experiment übereinstimmen würde.

4) Jede Art von Kollaps ist im vorliegenden Fall unnötig, wie man anhand von Mott‘s Berechnung erkennt. Diese Berechnung sind nach heutigen Verständnis übrigens extrem simpel, jeder Student mit Vorlesung QM 1 - also alle mit Diplom / Master - können sie verstehen, wenn da nicht (1) wäre.

Interpretationen wie Consistent Histories gehen genau in diese Richtung: axiomatische Präzisierung von „Kopenhagen“ als statistische Interpretation ohne Kollaps.

TomS
26.06.21, 13:00
Jetzt müssen wir aufpassen, nicht zu sehr über Leute im PF zu diskutieren.

Es liegt in der Natur der Sache, dass es bei den Interpretationen unterschiedliche Standpunkte gibt, ja geben muss. Genau das macht die Diskussion dort interessant.
Das ist nicht das Problem. Ich nenne mal einige Muster, was ich damit meine „dass die Leute nicht verstehen, dass man durchaus unterschiedliche Standpunkte jeweils einzeln objektiv betrachten muss“ und warum sich Diskussion dort aber auch anderswo so ermüdend gestalten.

1) Jede Interpretationen hat neben ihren Axiomen noch offene oder versteckte Grundhaltungen wie „agnostisch“, „realistisch“ oder „ontisch“. Diese können nicht weiter hinterfragt werden. Wenn ich nun eine Interpretation kritisiere, dann muss ich das unter Einhaltung ihrer Grundhaltung tun, andernfalls kritisiere ich nicht die Interpretation sondern die Grundhaltung. Man muss den Unterschied verstehen, und das ist oft nicht gegeben.

Bsp.: Wenn ich diskutieren möchte, in wie weit ein bestimmter Architekturstil das Ziel erreicht, Gott durch zum Himmel strebende Linien und lichterfüllte Räume zu loben, dann ist das Argumente, es gäbe keinen Gott oder man könne seine Existenz weder beweisen noch widerlegen, und deswegen könne man Gott in dieser Form nicht loben, schlichtweg Blödsinn. Es ist ebenfalls Blödsinn, zu behaupten, in kleinen Privatwohnungen oder Kapellen könne man Gott im persönlichen Gebet ebenfalls loben, weswegen gotische Sakralbauten diesbzgl. irrelevant wären.

Wenn die MWI einen ontischen Ansatz verfolgt, kann man die entweder innerhalb dieser Grundhaltung kritisieren, oder gar nicht. Wenn eine minimalistische Interpretation eine agnostische Grundhaltung einnimmt, dann kann man nicht kritisieren, dass sie nur stochastische Aussagen macht.

Wenn man nach hunderten von Posts immer noch nicht weiter ist, dann lässt man es besser bleiben.

2) Schauen wir uns mal drei Aussagen an:
Der Kern des quantenmechanischen Formalismus führt zur Vorhersage makroskopischer Superposition.
Die Kopenhagener Interpretation führt zusätzlich das Postulat eines Kollapses ein.
Die MWI gelangt aufgrund eines minimalen Axiomensystems sowie einer ontischen Grundhaltung zur Akzeptanz der realen Existenz vieler Welten.

Wenn sich Leute an Diskussionen beteiligen, die nicht akzeptieren können, dass dies ein sinnvoller Ausgangspunkt einer Diskussion ist und die den semantischen Unterschied der fett gedruckten Begriffe nicht verstehen, dann ist Hopfen und Malz verloren.

Wenn ich nach Anzeichen frage, die die eine oder andere Interpretation stützen, findest solche, die Richtung VWI tendieren.
Inwiefern? Meine gesamte Argumentation zielt darauf ab, dass ein Kollaps irrelevant ist. Das hat noch nichts mit MWI zu tun.

… während diese dort so kommentiert wird: " Decoherence has improved the understanding of the measurement problem, but only for the part which recognizes that it does not solve the measurement problem."
Wenn du dir den Satz nochmal durchliest, wirst du feststellen, dass er Quatsch ist.

Du weißt, dass ich die MWI bevorzuge, weil ich einen ontischen Ansatz verfolge. Ich wiederum weiß, dass du damit Probleme hast. Also ist meine Zielsetzung hier eine andere: zu zeigen, dass der Kollaps nicht nur überflüssig sondern hinderlich ist, um das Messproblem zu verstehen - auch für minimalistische, agnostische Ansätze.

Wenn du die kurze Arbeit zu Mott liest und feststellst, dass kein Kollaps benötigt wird, warum möchtest du dann daran festhalten? Wenn du dann qualitative Argumente liest, die mathematisch hinter Mott’s Arbeit zurückbleiben, nur um den Kollaps zu stützen, warum solltest du sie ernst nehmen? Inwiefern führt Mott’s Argumentation hin zur MWI oder tendiert zu dieser? In dem Artikel steht dazu nichts.

Ich rede hier nicht pro MWI sondern contra Kollaps. Wenn man auf den Kollaps verzichtet und eine ontische Grundhaltung einnimmt, dann und nur dann leitet bereits Mott’s Arbeit sowie die Dekohärenz hin zur MWI. Ich habe aber weiter oben schon geschrieben, dass du diesen ontischen Schritt nicht machen musst. Deswegen bleibt die Mathematik ohne Kollaps dennoch richtig und epistemisch / experimentell überprüfbar zutreffend.

Timm
26.06.21, 17:36
Wenn du die kurze Arbeit zu Mott liest und feststellst, dass kein Kollaps benötigt wird, warum möchtest du dann daran festhalten? Wenn du dann qualitative Argumente liest, die mathematisch hinter Mott’s Arbeit zurückbleiben, nur um den Kollaps zu stützen, warum solltest du sie ernst nehmen? Inwiefern führt Mott’s Argumentation hin zur MWI oder tendiert zu dieser? In dem Artikel steht dazu nichts.

Ich werde diesen https://arxiv.org/pdf/1209.2665.pdf Artikel nicht durcharbeiten. Die Wortsuche ergibt keinen Treffer für "collapse". Sollte sie aber, wenn eine der Aussagen die ist, dass beim Messprozess kein Kollaps stattfindet.

Wenn dieser Artikel das tatsächlich erhärtet, sollte das enorme Beachtung gefunden haben, sollte es viele Folgeartikel geben, die sich damit beschäftigen und sollte denjenigen, die am Kollaps festhalten schon längst das Wasser abgegraben sein. Falls nicht, kommt die Frage nach der mangelnden Akzeptanz, bzw. nach der Interpretation dieses Artikels.

TomS
26.06.21, 23:11
Ich werde diesen https://arxiv.org/pdf/1209.2665.pdf Artikel nicht durcharbeiten. Die Wortsuche ergibt keinen Treffer für "collapse". Sollte sie aber, wenn eine der Aussagen die ist, dass beim Messprozess kein Kollaps stattfindet.
So langsam frage ich mich, ob ich nicht doch meine Zeit verschwende.

Der Artikel enthält ca. zwanzig mal den Begriff „reduction“ im Kontext „wave packet reduction“, „reduction of the probability packet“, „reduction postulate“.

Wenn dieser Artikel das tatsächlich erhärtet, sollte das enorme Beachtung gefunden haben, sollte es viele Folgeartikel geben, die sich damit beschäftigen und sollte denjenigen, die am Kollaps festhalten schon längst das Wasser abgegraben sein. Falls nicht, kommt die Frage nach der mangelnden Akzeptanz, bzw. nach der Interpretation dieses Artikels.
Von nichts anderem rede ich.

Timm
27.06.21, 09:00
Der Artikel enthält ca. zwanzig mal den Begriff „reduction“ im Kontext „wave packet reduction“, „reduction of the probability packet“, „reduction postulate“.

Ist es diese Passage, auf die du dich beziehst?

A further point to be emphasized concerns the relevance of the result obtained in the approach b). It is shown that, under the right conditions and with high probability, one can only observe straight tracks. This is a highly non trivial result obtained exploiting Schrödinger dynamics without having any recourse to the wave packet reduction rule. The reduction should only be invoked at the stage of the ”observation of the actual track” described by the α-particle. It is remarkable that, as we have seen in section 3, this aspect was understood by Darwin even before the quantitative analysis performed by Mott.

Es ist die einzige, bei der ich eine gewisse Relevanz bzgl. deiner Aussage (Kollaps unnötig) erkenne. Vielleicht hast du das schon mal zitiert, dann wäre es mir entgangen.

TomS
27.06.21, 09:42
Hier kann man das ursprüngliche Paper von Mott lesen:

https://royalsocietypublishing.org/doi/pdf/10.1098/rspa.1929.0205

Mott kümmert sich nicht um einen Kollaps o.ä. sondern löst das Problem direkt mittels Schrödingergleichung und Bornscher Regel. Jedem Leser wird klar, dass dies offensichtlich ohne Kollaps funktioniert.

Warum er das nicht explizit erwähnt, kann ich nicht sagen, ich habe jedoch eine Vermutungen: Mott schreibt, dass zu zeigen ist, wie teilchenartige Phänomene aus der Wellenmechanik abgeleitet werden können. Das war zu der damaligen Zeit sicher die essentielle physikalische Frage, deren mögliche direkte Lösung Bohr im Zuge der Komplementarität explizit verneint hat; ihm zufolge verhält sich oder erscheint das Quantenobjekt entweder als Welle oder als Teilchen. Mott zufolge versteht man teilchenartiges Verhalten jedoch tatsächlich mittels Wellenphänomen, nämlich durch die fortwährende Lokalisierung aufgrund der Streuung an Atomen, d.h. ohne Zusatzannahmen - egal welche.

Warum sollte Mott also eine völlig irrelevante zweite Baustelle bzgl. eines technischen Details aufmachen? Mott liefert eine minimalistische mathematische Erklärung der Spuren in einer Nebelkammer, die gleichzeitig die Komplementarität falsifiziert; sein Paper ist auch so ein Frontalangriff auf ein Dogma der Kopenhagener Schule.



Es ist auch keineswegs so, dass ein geeignet umformulierter Kollaps unzulässig wäre - s.o.; er ist jedoch explizit künstlich und überflüssig. Wenn Mott an Physik und nicht an Philosophie liegt, ignoriert er letzteres.

Zur Frage, wie dieser Kollaps eingeführt werden kann: Mott zeigt, dass ein Zustand

|u1, u2, f>

mit zweimaliger Streuung, wobei u1, u2 die beiden Atome und f das Alphateilchen beschreibt, eine Struktur aufweist, in der f nur in einem engen Vorwärtskegel ausgehend von a2 in Richtung a2-a1 nicht verschwindet.

Sei E(a2) ein Projektor auf einen Bereich des Ortsraumes um a2. Dann gilt für den von a2 aus auslaufenden Zustand in extrem guter Näherung

E(a2) |f> = |f>

da E(a2) lediglich Komponenten des Zustandes abschneidet, die aufgrund der Struktur von |f> ohnehin in extrem guter Näherung verschwinden. Die Projektion wird also so eingeführt, dass sie Komponenten eliminiert, die ohnehin irrelevant sind.

Ein derartiger Kollaps - es ist nicht der im Sinne von Neumanns sondern eine modifizierte Fassung - ist also gleichermaßen harmlos und möglich wie überflüssig. Er trägt mathematisch und physikalisch nichts bei, er zeigt lediglich, dass man weiterhin im Sinne von Kopenhagen denken darf, wenn man möchte. Er erklärt jedoch nicht, warum man das tun sollte, da er - ex post konstruiert - keine Vorhersagekraft hat.

Letztlich ist dies die Aussage auch jeder modernen non-collapse Interpretation.

TomS
27.06.21, 09:55
Ist es diese Passage, auf die du dich beziehst?

A further point to be emphasized concerns the relevance of the result obtained in the approach b). It is shown that, under the right conditions and with high probability, one can only observe straight tracks. This is a highly non trivial result obtained exploiting Schrödinger dynamics without having any recourse to the wave packet reduction rule. The reduction should only be invoked at the stage of the ”observation of the actual track” described by the α-particle. It is remarkable that, as we have seen in section 3, this aspect was understood by Darwin even before the quantitative analysis performed by Mott.

Es ist die einzige, bei der ich eine gewisse Relevanz bzgl. deiner Aussage (Kollaps unnötig) erkenne. Vielleicht hast du das schon mal zitiert, dann wäre es mir entgangen.

Es hat nicht „eine gewisse Relevanz“, es ist der zentrale Punkt!

Ich verstehe nicht, warum du erwartest, dass Mott auf etwas herumreitet, was er einmal explizit schreibt und was allen Adressaten offensichtlich klar sein musste. Das Problem wurde von Einstein und Heisenberg diskutiert; sicher war auch Bohr involviert. Soll Mott - damals noch nicht mal 25, kurz vor oder nach seiner Ernennung zum Privatdozenten, international unbekannt - in einer seiner ersten Veröffentlichungen ständig wiederholen, dass der große Bohr völlig daneben liegt? Es ist doch nur guter Stil und Understatement, es dem Leser zu überlassen, dies zu erkennen.

EDIT - noch etwas: Gerade der knappe, präzise und minimalistische Stil an sich hat bereits enorme Wirkung gegen teils ausufernde und wolkige Darstellungen von Bohr sowie aus seinem Umfeld. Mott zeigt auf fünf Seiten, dass Komplementarität unnötig ist, dass also teilchenartiges Verhalten direkt mittels der Wellenmechanik verstanden werden kann, dass das „Messgerät“ Nebelkammer direkt quantenmechanisch modelliert werden kann und dass all dies ohne Projektionspostulat gelingt.

Timm
27.06.21, 15:36
Es hat nicht „eine gewisse Relevanz“, es ist der zentrale Punkt!

Dann ist es ja gut, dass wir nach tagelangem hin und her endlich zu ihm vorgestoßen sind. An dieser Stelle ein Vorschlag. Statt zu sehr ins Detail zu gehen, könntest du abstract-artig das Wesentliche herausschälen. Dass du sie kennst, musst du nicht beweisen. Ich aber bin mehr an den Ergebnissen interessiert. Wenn ich mehr wissen will, würde ich mich schon melden.

Das sogenannte "Messproblem" betrachtet, ob und wie der Kollaps der Wellenfunktion stattfindet. Demnach liefern Figari&Teta einen bedeutsamen Beitrag hierzu, mehr noch, das Messproblem wäre vom Tisch. Ist dieses paper schlicht in Vergessenheit geraten, wurde es in der community nicht ernst genommen ... was weißt du darüber?

TomS
27.06.21, 15:56
Ich würde gerne mit den zentralen Aussagen Bohrs et al. beginnen und das Paper von Mott zusammenfassen, wenn es dir recht ist. Damit hast du automatisch einen Teil der Zusammenfassung des neueren Papers.

Dabei kommt jedoch nichts raus, was nicht bereits hier im Thread steht ;-)

Timm
27.06.21, 16:32
Kannst du erst mal auf die Frage im 2. Absatz eingehen?

Zusammenfassung gerne ohne technische Details.

TomS
27.06.21, 17:44
Das Paper von Mott ist nicht in Vergessenheit geraten, es wurde und wird immer wieder im Kontext Decoherence, Consistent Histories u.a. als frühe Arbeit in diese Richtung zitiert.


Tatsache ist aber, dass Alternativen zu Kopenhagen sowie generell Arbeiten zu Grundlagen zur Quantenmechanik über Jahrzehnte nicht gerne gesehen waren; von Bohm, Bell über Everett bis Zeh bekamen Physiker bis in die 60er Jahre vereinzelt Gegenwind bei Publikationen in namhaften Zeitschriften oder wurden von der großen Masse schlicht ignoriert. Man befasste sich nicht in der Breite mit diesen Themen; das war keine „ernsthafte Physik“.

Die Situation änderte sich erst langsam in den 60ern aufgrund der Beschäftigung mit der nicht-perturbativen Quantengravitation (Wheeler–DeWitt equation); diese lässt eben keinen externen, klassischen Beobachter zu. Wheeler hatte Everett’s Arbeit in den 50ern noch begrifflich „entschärft“, deWitt prägte den Begriff der „vielen Welten“ in den 60ern oder frühen 70ern. Die Decoherence (Zeh, 1969) und mit ihr die Wiederentdeckung der Ergebnisse Everett’s blieb nochmal zehn Jahre liegen (Zurec, 1981). Die Consistent Histories stammt ebenfalls aus den 80ern.

Eigene Erfahrung: bei einem Seminar ca. 1990 wurde ein Vortrag über die Wheeler–DeWitt Gleichung eher belächelt, der darauf folgende über Many-Worlds als unphysikalisch ignoriert. In den QM I + II Vorlesungen wurden die Themen ignoriert, Seminare dazu gab es an drei Unis über zehn Jahre (Studium und später) keines. Bei der Vorbereitung eines Kongresses zu modernen Methoden der QCD Anfang der 90er diskutierte ich mit meinen Prof. über die Speaker; es waren sehr bekannte Namen dabei. Auf meine Frage, an was ‘t Hooft z.Zt. arbeiten würde meinte mein Prof. spöttisch „Quantengravitation; und Grundlagen der Quantenmechanik“; ‘t Hooft war nicht eingeladen.

Wenn du dir heute Vorlesungsskripte anschaust, findest du viele, die die Diskussion der vergangenen Jahrzehnte schlicht ignorieren. Generationen von Physikern war das Messproblem sch…egal, und so haben sie das auch weitergegeben. Teilweise ist das heute noch so.

Ok? Dann können wir zum Abstract kommen?

Timm
27.06.21, 19:41
Warte bitte mal ab, ich muss da etwas überdenken und gebe dir Morgen Bescheid.

TomS
27.06.21, 21:16
Ok, gerne.


Beispiel zum oben gesagten: Google-Suche, dritter Treffer:

http://www-thphys.physics.ox.ac.uk/people/FabianEssler/QM/QM2019.pdf
Lecture Notes for Quantum Mechanics
F.H.L. Essler
The Rudolf Peierls Centre for Theoretical Physics Oxford University, Oxford OX1 3PU, UK
February 17, 2021

TomS
28.06.21, 09:32
Bevor ich Mott’s Arbeit zusammenfasse, sollte man sie im zeitlichen Kontext einordnen. Ich habe dazu noch etwas gesucht und diesen Übersichtsartikel zu Tage gefördert:

bacciagaluppi_preprint.pdf (mpg.de)
The statistical interpretation according to Born and Heisenberg
Guido Bacciagaluppi
Abstract:
At the 1927 Solvay conference Born and Heisenberg presented a joint report on quantum mechanics. I suggest that the significance of this report lies in that it contains a ‘final’ formulation of the statistical interpretation of quantum mechanics that goes beyond Born’s original proposal. In particular, this formulation imports elements from Heisenberg’s work as well as from the transformation theory of Dirac and Jordan. I suggest further a reading of Born and Heisenberg’s position in which the wave function is an effective notion. This can make sense of a remarkable aspect of their presentation, namely the fact that the ‘quantum mechanics’ of Born and Heisenberg apparently lacks wave function collapse.

Der letzte Satz ist interessant, da hier die Argumentation Heisenbergs eher andersherum gelesen wird; der Kollaps wird nicht zwingend benötigt.

Zunächst Born “pro Kollaps”:

.... one must appeal to the notion of ‘reduction of the probability packet’ developed by Heisenberg. The description of the emission by a spherical wave is valid only for as long as one does not observe ionization; as soon as such ionization is shown by the appearance of cloud droplets, in order to describe what happens afterwards one must ‘reduce’ the wave packet in the immediate vicinity of the drops. One thus obtains a wave packet in the form of a ray, which corresponds to the corpuscular character of the phenomenon.
(Born)

Pauli kritisiert den Kollaps als “ad hoc”:

This is precisely a point that was not quite satisfactory in Heisenberg [(1927)]; there the ‘reduction of the packets’ seemed a little mystical. Now in fact it should be stressed that such reductions are not necessary in the first place if one includes in the system all means of measurement. But in order to describe at all observational results theoretically, one has to ask what one can say alone about a part of the whole system. And then from the complete solution one sees immediately that, in many cases (of course not always), leaving out the means of observation can be formally replaced by such reductions.
(Pauli)

Pauli geht also davon aus, dass der Kollaps zunächst ebenfalls kein unverzichtbares Element der Theorie sein muss, wenn das Messgerät als Teil des Gesamtsystems aufgefasst wird. Das ist letztlich genau die Sichtweise vieler moderner Theorien, die Bohr wiederum explizit abgelehnt hat; ihm zufolge muss das Messgerät immer klassisch betrachtet werden. Nach Pauli tritt ein „effektiver Kollaps“ dann auf, wenn man Quantensystem und Messgerät bzw. Beobachtung getrennt betrachtet; so kann man übrigens auch von Neumann lesen.

Mr Pauli has asked me if it is not possible to describe the process without the reduction of wave packets, by resorting to a multi-dimensional space whose number of dimensions is three times the number of all the particles present .... . This is in fact possible and can even be represented in a very anschaulich manner [d’une mani`ere fort intuitive] by means of an appropriate simplification, but this does not lead us further as regards the fundamental questions.
(Born)

Man beachte, dass diese Diskussionen sämtlich vor der ersten vollständigen Berechnung Mott’s geführt wurden.

Zunächst ist anzumerken, dass es 1927 und später wohl keine klare Haltung bzgl. des Kollapses gab.

Es deutet sich jedoch durchaus an, dass eine minimalistische, stochastische Interpretation ohne Kollaps- bzw. Projektionspostulat möglich ist. Warum sich diese Sichtweise nicht durchsetzen konnte, kann ich nur vermuten, möglicherweise aufgrund des sehr erfolgreichen späteren Buches von Neumanns – Mathematical Foundations of Quantum Mechanics, 1932 – oder weil man meinte, eine teilchenartige und zugleich realistische (ontische) Interpretation sei ausschließlich mit Kollaps zu erreichen.

Anyway, so wird es einfacher, zu zeigen, worauf Mott hinauswollte, und dass er damit keineswegs gegen Kopenhagen als geschlossenes Ganzes stand.

Timm
28.06.21, 15:42
Es kann sein, dass ein Mißverständnis vorliegt, jedenfalls bin ich nicht sicher, ob ich dich richtig verstanden habe.

Nochmal zurück zu dem Zitat aus dem verlinkten Artikel.

Zitat von Timm
Ist es diese Passage, auf die du dich beziehst?

A further point to be emphasized concerns the relevance of the result obtained in the approach b). It is shown that, under the right conditions and with high probability, one can only observe straight tracks. This is a highly non trivial result obtained exploiting Schrödinger dynamics without having any recourse to the wave packet reduction rule. The reduction should only be invoked at the stage of the ”observation of the actual track” described by the α-particle. It is remarkable that, as we have seen in section 3, this aspect was understood by Darwin even before the quantitative analysis performed by Mott.

Es ist die einzige, bei der ich eine gewisse Relevanz bzgl. deiner Aussage (Kollaps unnötig) erkenne. Vielleicht hast du das schon mal zitiert, dann wäre es mir entgangen.

Es hat nicht „eine gewisse Relevanz“, es ist der zentrale Punkt!

Hier hatte ich dich so verstanden, dass du den Befund bzgl. der geraden Linie verallgemeinerst. Das spezielle Beispiel zeigt, dass die Annahme eines Kollapses der WF zur Erklärung der geraden Linie nicht benötigt wird und dies ganz generell gilt, sozusagen pars pro toto. So hatte ich dich verstanden. Kollaps Interpretationen adieu.

Habe ich dich missverstanden?

TomS
28.06.21, 16:19
Es kann sein, dass ein Mißverständnis vorliegt, jedenfalls bin ich nicht sicher, ob ich dich richtig verstanden habe.
Vermutlich.

Das spezielle Beispiel zeigt, dass die Annahme eines Kollapses der WF zur Erklärung der geraden Linie nicht benötigt wird und dies ganz generell gilt, sozusagen pars pro toto. So hatte ich dich verstanden. Kollaps Interpretationen adieu.
Wenn du mit 'pars pro toto' meinst, dass die vorliegende Arbeit von Mott völlig allgemeingültig zeigt, dass der Kollaps nie benötigt wird - bei keinem anderen System - dann hättest du mich missverstanden; die konkrete Arbeit von Mott zeigt das konkret für diesen einen Fall, mehr nicht.

Nur -
1) inzwischen gibt es viele andere Arbeiten, die dies für große Klassen weiterer Systeme zeigen
2) Interpretationen, die einen Kollaps postulieren, postulieren ihn als fundamentale, allgemeingültige Zutat

Natürlich kann keine physikalische Überlegung - theoretisch oder praktisch - den Kollaps für beliebige Systeme ein für allemal ausschließen. Aber (1) konkrete Berechnungen können dies für konkrete Systeme tun - und tun dies - und damit ist (2) sicher falsch.

Die einzig verbleibende Möglichkeit wäre, dass neben der großen Klasse von Systemen, bei denen gezeigt wurde, dass kein Kollaps benötigt wird, einzelne Systeme existieren, für dies nicht zutrifft. Nur erstens behauptet das niemand so, und zweitens existiert dazu kein Beispiel.

Zusammenfassend
1) für eine große Klasse von Systemen kann die Notwendigkeit eines Kollapses heute sicher ausgeschlossen werden - unabhängig von dem, was man so liest - und Mott's Arbeit ist wohl das erste Beispiel dazu
2) Gegenbeispiele, in denen ein Kollaps als fundamentales Postulat sicher benötigt wird, sind nicht bekannt

Damit erscheint die Quantenmechanik ohne Kollapspostulat als ganz normale Theorie, die es zu überprüfen gilt - so wie z.B. die Elektrodynamik oder die Relativitätstheorie auch (diese Aussage bezieht sich natürlich ausschließlich auf die Berechnung konkreter und experimentell überprüfbarer Vorhersagen, nicht jedoch auf Interpretationen)

Ja, Kollaps-Interpretationen adieu - solange niemand ein Gegenbeispiel findet; Maxwellsche Gleichungen zutreffend, solange niemand ein Gegenbeispiel findet; Lichtgeschwindigkeit = universelle Grenzgeschwindigkeit, solange niemand ein Gegenbeispiel findet.

Es ist die einzige, bei der ich eine gewisse Relevanz bzgl. deiner Aussage (Kollaps unnötig) erkenne. Vielleicht hast du das schon mal zitiert, dann wäre es mir entgangen.
Ich hatte das noch nicht zitiert.

Ich hatte eine gewisse Relevanz hervorgehoben, weil sowohl Mott's Paper als auch die späteren Darstellungen seiner Arbeit nicht eine "gewisse Relevanz" haben, dass im Falle der Nebelkammer kein Kollaps benötigt wird, sondern dass bereits Mott's Arbeit dies für den Falle der Nebelkammer präzise, einfach und vollumfänglich zeigt. Es gibt nach Mott's Arbeit nicht den Hauch eine Grundes, für die Berechnung der Beobachtungen in der Nebelkammer ein Kollaps einzuführen.

Timm
28.06.21, 21:28
Wenn du mit 'pars pro toto' meinst, dass die vorliegende Arbeit von Mott völlig allgemeingültig zeigt, dass der Kollaps nie benötigt wird - bei keinem anderen System - dann hättest du mich missverstanden; die konkrete Arbeit von Mott zeigt das konkret für diesen einen Fall, mehr nicht.
Ich habe dich deshalb missverstanden, weil ich dein statement

4) Jede Art von Kollaps ist im vorliegenden Fall unnötig, wie man anhand von Mott‘s Berechnung erkennt.

für in Stein gemeißelt hielt. Nun rückst du das zurecht, was diesen Punkt klärt.

2) Gegenbeispiele, in denen ein Kollaps als fundamentales Postulat sicher benötigt wird, sind nicht bekannt
Weshalb bietet die folgende Aussage kein Gegenbeispiel:

Ein einziges definiertes Messergebnis eines verschränkten Systems erklärt man abhängig von der Interpretation mit dem Kollaps der WF oder mit den VW.

TomS
28.06.21, 22:03
Ich habe dich deshalb missverstanden, weil ich dein statement

„Jede Art von Kollaps ist im vorliegenden Fall unnötig, wie man anhand von Mott‘s Berechnung erkennt.“

für in Stein gemeißelt hielt. Nun rückst du das zurecht, was diesen Punkt klärt.
Gut, dass du das so explizit sagst, denn ich merke, dass der Punkt nicht geklärt ist.

Es gibt zwei Dimension:
a) welches System?
b) welche Art von Kollaps?

Welches System? Im vorliegenden Fall der Nebelkammer ist jede Art von Kollaps unnötig, wie man anhand von Mott‘s Berechnung erkennt. Hierbei geht es um genau ein System. Es mag andere Systeme geben, für die ein Kollaps notwendig sein könnte; mir sind jedoch keinerlei Experimente bekannt, die dies in irgendeiner Form nahelegen.

Welche Art von Kollaps? Die nach von Neumann in einen Ortseigenzustand können wir ausschließen, sie liefert falsche Vorhersagen. Die im Sinne eines „weak measurements“ könnten wir einführen - s.o. - dazu müssen aber andere Beweggründe herangezogen werden als nur die experimentellen Fakten. Eine andere Art von Kollaps zur Aufrechterhaltung einer ontischen Sichtweise wäre genau so ein anderer Grund.

Nur zur Klarstellung, ich rücke nicht von meiner ontischen Sichtweise ab, ich stelle die lediglich hinten an; ohne diese ist ein Kollaps unnötig. Mott‘s Berechnung zeigt, dass ausschließlich für gerade Spuren eine Wahrscheinlichkeit größer Null vorliegt. Mehr braucht ein Positivist nicht.

Weshalb bietet die folgende Aussage kein Gegenbeispiel:
Ein einziges definiertes Messergebnis eines verschränkten Systems erklärt man abhängig von der Interpretation mit dem Kollaps der WF oder mit den VW.
Das liefert wiederum nur dann ein Gegenbeispiel, wenn du eine ontische Interpretation wünschst.

Timm
29.06.21, 08:34
Das liefert wiederum nur dann ein Gegenbeispiel, wenn du eine ontische Interpretation wünschst.
Ich wünsche gar nichts. Ob ontisch oder nicht ist eine Frage Interpretation.

Ich stelle lediglich fest, dass diese Aussage


2) Gegenbeispiele, in denen ein Kollaps als fundamentales Postulat sicher benötigt wird, sind nicht bekannt

irreführend ist. Gegenbeispiele sind bekannt, sofern man Interpretationsoffen argumentiert.

Davon abgesehen halte ich die Wortwahl "Gegenbeispiel" für nicht zielführend. Es gibt Kollaps Interpretationen und Nicht-Kollaps Interpretationen, fertig. Daran hat sich seit Mott bis heute nichts geändert.

Wäre mir das angesprochene Missverständnis schon vor ein paar Tagen aufgefallen, wäre dieser Thread erheblich kürzer geraten. :D

TomS
29.06.21, 09:05
Ich stelle lediglich fest, dass diese Aussage "Gegenbeispiele, in denen ein Kollaps als fundamentales Postulat sicher benötigt wird, sind nicht bekannt" irreführend ist. Gegenbeispiele sind bekannt, sofern man Interpretationsoffen argumentiert.
Ja, sofern man interpretationsoffen argumentiert.

Aber darum geht es mir zunächst nicht, und daher ich schreibe explizit direkt nach dem von dir kritisierten Satz: "Damit erscheint die Quantenmechanik ohne Kollapspostulat als ganz normale Theorie, die es zu überprüfen gilt - so wie z.B. die Elektrodynamik oder die Relativitätstheorie auch (diese Aussage bezieht sich natürlich ausschließlich auf die Berechnung konkreter und experimentell überprüfbarer Vorhersagen, nicht jedoch auf Interpretationen)".

In diesem Sinne benötige ich sicher keinen Kollaps, solange diese Theorie (Hilbertraum, Schrödingergleichung, Bornsche Regel, kein Projektionspostulat) durch kein Experiment falsifiziert ist.

Davon abgesehen halte ich die Wortwahl "Gegenbeispiel" für nicht zielführend. Es gibt Kollaps Interpretationen und Nicht-Kollaps Interpretationen, fertig. Daran hat sich seit Mott bis heute nichts geändert.
Letzteres stelle ich doch überhaupt nicht in Frage. Alles was ich sage ist, dass kein Gegenbeispiel bekannt ist, das den o.g. Formalismus (Hilbertraum, Schrödingergleichung, Bornsche Regel, kein Projektionspostulat) dahingehend falsifizieren würde, dass er experimentell überprüfbare Vorhersagen macht, die unzutreffend wären, so dass daraus abgeleitet werden könnte, dass der Kollaps als fundamentales Postulat zur korrekten Berechnung benötigt wird.

Das heißt, das Kollaps-Interpretationen möglich und zulässig sind, dass sie jedoch nicht alleine aus den experimentellen Vorhersagen motiviert werden können.

TomS
29.06.21, 10:43
Ich denke, es ist an der Zeit, eine kurze Zusammenfassung von Mott‘s Arbeit einzustellen [sowie weitere Anmerkungen in eckiger Klammer, die bei Mott nicht explizit ausgeführt sind, sich jedoch unmittelbar erschließen lassen, teilweise jedoch erst in der zeitlichen Rückschau]. Damit haben wir dann hoffentlich zumindest für den Fall der Nebelkammer Klarheit.


Mott zeigt, wie die typische teilchenartige Eigenschaft – Spuren in einer Nebelkammer – aus der Wellenmechanik sowie unter Anwendung der Bornschen Regel abgeleitet werden kann; ersteres bedeutet hier die stationäre Schrödingergleichung in mindestens zweiter Ordnung Störungstheorie, d.h. für mindestens zwei Streuungen.

Zunächst wäre zu erwarten, dass eine vom Zentrum auslaufende Kugelwelle zu isotropen Phänomenen führt. Es zeigt sich jedoch, dass bei zwei- oder mehrfacher Streuung rein mathematisch eine „isotrope Superposition über – jeweils für sich gesehen anisotrope – Komponenten der Wellenfunktion“ resultiert. Jede einzelne anisotrope d.h. gerichtete Komponente korrespondiert zu einer einzelnen Spur.

Laut Mott ist zur Berechnung weder eine teilchenartige Interpretation notwendig noch eine „Reduktion eines Wellenpaketes“ je einzelner Streuung; erstere wird von ihm explizit ausgeschlossen, letztere weder erwähnt noch verwendet, und ist daher offensichtlich für die Herleitung des Ergebnisses – isotrope Superposition anisotroper gerichteter Komponenten – mathematisch verzichtbar [und wäre in der häufig präsentierten einfachen Fassung des Projektionspostulates im Falle der Nebelkammer sogar explizit falsch].

In der konkreten Berechnung zeigt Mott, dass eine Komponente der Gesamtwellenfunktion nach zweifacher Streuung für die beiden ionisierten Atome sowie das auslaufende alpha-Teilchen eine Form aufweist, in der die Wellenfunktion des alpha-Teilchens lediglich innerhalb eines extrem schmalen Kegels in Vorwärtsrichtung – definiert durch die Verbindungslinie der Streuzentren – eine merklich von Null verschiedene Amplitude aufweist. Jede Streuung erzeugt demnach eine im Orts- und Impulsraum eng lokalisierte und daher gerichtete Komponente der Gesamtwellenfunktion.

Dies passt zu den experimentell beobachtbaren Spuren: die Wahrscheinlichkeit, mehrere nicht auf einer Geraden liegende Streuzentren (ionisierter Atome und damit Tröpfchen) zu beobachten, verschwindet in extrem guter Näherung.


[Eine Gegenposition zu Kopenhagen ist an zwei Stellen explizit erkennbar: das makroskopische Phänomen kann im Gegensatz zur Meinung Bohrs tatsächlich rein quantenmechanisch behandelt werden; ein teilchenartiges Phänomen ist im Widerspruch zur Behauptung des Welle-Teilchen-Dualismus rein auf Basis der Wellenmechanik ableitbar.

Das Messproblem - damals nicht so bezeichnet - wird für den Fall der Nebelkammer und speziell im Rahmen einer minimalistischen, rein statistischen Interpretation gelöst: die Emergenz eines makroskopischen Phänomens gerichteter teilchenartiger Spuren resultiert ausschließlich aus wiederholter Streuung der Wellenfunktion im Rahmen der unitären Dynamik unter Verzicht auf das Projektionspostulat.

An sich folgt aus Mott’s Arbeit lediglich die Form der Wellenfunktion für zweimalige Streuung; eine iterierte Anwendung führt jedoch sofort auf „makroskopisch strahlenartige Komponenten“. Durch wiederholte Streuung und damit verbunden Lokalisierung werden die Komponenten der Wellenfunktion zu engen Zylindern fokussiert.

Die Struktur der „isotropen Superposition jeweils für sich gesehen anisotroper Strahlen“ entspräche in einer modernen Lesart der Superposition einzelner Komponenten gemäß der Dekohärenz.

Aus der Struktur des Endzustandes kann eine Schar geeigneter Projektionsoperatoren konstruiert werden. Tatsächlich werden diese implizit im Rahmen der Bornschen Regel auch angewandt, spielen jedoch keine Rolle im Sinne des Projektionspostulats, das insbs. im Zuge der Berechnung des Endzustandes zu verwenden wäre, jedoch nicht verwendet wird. Da die Schar der Projektionsoperatoren außerdem erst aus einer konkreten Rechnung für ein spezielles System folgt, kann sie nicht a priori d.h. axiomatisch vorgegeben werden. Dies entspricht im Wesentlichen der Sichtweise der Dekohärenz.

Es liegt eine minimalistische, rein statistische Interpretation ohne ontischen Anspruch vor. Insbs. wird nicht darauf eingegangen, wie konkret eine bestimmte tatsächlich realisierte Spur aus der Gesamtheit aller möglichen Spuren hervorgeht. Die Argumentation passt damit auch zu modernen, stochastischen non-collapse-Interpretationen.

Da Mott sämtliche experimentell überprüfbaren Vorhersagen ohne Rückgriff auf ein (geeignet formuliertes) Projektionspostulates ableitet, bedarf die Einführung desselben einer darüberhinausgehenden Motivation. Die häufig genannte Begründung wiederholter Messungen derselben Observablen greift im vorliegenden Fall mehrfacher Streuung offenbar nicht.]

Timm
30.06.21, 09:06
Ja, sofern man interpretationsoffen argumentiert.

Aber darum geht es mir zunächst nicht, und daher ich schreibe explizit direkt nach dem von dir kritisierten Satz: "Damit erscheint die Quantenmechanik ohne Kollapspostulat als ganz normale Theorie, die es zu überprüfen gilt - so wie z.B. die Elektrodynamik oder die Relativitätstheorie auch (diese Aussage bezieht sich natürlich ausschließlich auf die Berechnung konkreter und experimentell überprüfbarer Vorhersagen, nicht jedoch auf Interpretationen)".

Letzteres stelle ich doch überhaupt nicht in Frage.
Danke für die Klarstellungen.

Die Diskussion war interessant und insbesondere dein Eingehen auf das "Mott-Problem", dessen Auswirkung auf die Interpretationen ich zunächst überschätzt hatte. Die Details hierzu interessieren mich weniger.

TomS
30.06.21, 10:57
Danke für die Klarstellungen.

Die Diskussion war interessant und insbesondere dein Eingehen auf das "Mott-Problem", dessen Auswirkung auf die Interpretationen ich zunächst überschätzt hatte.
Das Mott-Problem (u.a. ähnlich gelagerte) kann man eigentlich kaum überschätzen, da es zeigt, dass viele Physiker seit Jahrzehnten evtl. einem Phantom hinterherjagen und sich dessen nicht mal bewusst sind.

Wo stehst du denn inzwischen mit deiner persönlichen Einschätzung der Interpretationen? Welche Gründe würdest du für eine Kollaps-Interpretation anführen? welche für Nicht-Kollaps-Interpretationen? (letzteres ohne MWI, das wäre nochmal eine völlig andere Baustelle)

Das würde mich nach dieser langen Diskussion schon interessieren.

Timm
30.06.21, 17:17
Als ich vor nun schon längerer Zeit einiges von Zeh gelesen hatte und auch eine kurze Korrespondenz mit ihm hatte, bin ich vorübergehend ein wenig in den Sog der VW geraten. Die Plausibilität schien mir kaum angreifbar. Die Wende zurück kam dann, nachdem ich Zeilinger's "Einstein's Schleier" gelesen hatte.

Meine Haltung ist jedoch agnostisch. Ich maße mir nicht an, die Wahrheit zu kennen und habe für missionarischen Eifer nichts übrig.

TomS
30.06.21, 19:55
Als ich vor nun schon längerer Zeit einiges von Zeh gelesen hatte und auch eine kurze Korrespondenz mit ihm hatte, bin ich vorübergehend ein wenig in den Sog der VW geraten. Die Plausibilität schien mir kaum angreifbar. Die Wende zurück kam dann, nachdem ich Zeilinger's "Einstein's Schleier" gelesen hatte.
Beide sind m.N.n. nicht die klarsten Vertreter der jeweiligen Sichtweise.

Meine Haltung ist jedoch agnostisch.
Ok. Trotzdem kannst du ja rein pragmatisch die etablierte Mathematik einschätzen und ihre Vorhersagen bewerten.

Für mich wäre ein Kollaps dann nicht notwendig, allenfalls in bestimmten Fällen praktisch, um zu korrekten Vorhersagen zu gelangen. Es gewinnt das schlankere Axiomensystem.

Jakito
02.07.21, 13:20
Für mich persönlich haben die isolierten und unzusammenhängenden Darstellungen von Zeh trotzdem am meisten Überzeugungsarbeit geleistet, VW und Dekohärenz ernst zu nehmen. Er hat halt klar die Probleme aufgezeigt, erklärt wieso die triviale Lösung zu kurz greift, und auch bei der von ihm bevorzugten Lösung nicht die Probleme verschwiegen. (Zeilinger habe ich nie gelesen.)

Bei Zeh habe ich zum ersten Mal gelesen, dass man für die VW mehr braucht, als nur einen Hilbertraum und einen Hamiltonian. (Er hat an dieser Stelle nicht explizit gesagt, welche der Optionen für das "Mehr" er bevorzugt. Er hat noch nichtmals versucht, die sinnvollen Optionen aufzuzählen. Das war zwar seltsam, aber dem tatsächlichen Verständnis eher förderlich.)

Allerdings habe ich mich jetzt entschlossen, diesem Diskussionsforum beizutreten, und in diesem Thread zu schreiben, weil mir aufgefallen ist, dass Timm derselbe ist, der auf PhysicsForums den entsprechenden Thread losgetreten hat, und mit dem ich auch auf BackReAction einen quasi unfertigen Austausch (http://backreaction.blogspot.com/2021/05/what-did-einstein-mean-by-spooky-action.html?showComment=1620766677814#c26020493652 09694346) hatte.

Aus verschiedenen Gründen habe ich da inzwischen stärker nachgeforscht, und festgestellt, dass wohl Sean Carroll in seinem Buch versehentlich die VW Interpretation in einer angreifbaren Version dargestellt hat. (Die Welt wird zu oft, und in zu viele Welten gespalten.) Diese Version ähnelt eher Bryce DeWitts Version als Everetts Version. Und deshalb ist dieser Angriff in der Folge dann auch passiert, von Sabine Hossenfelder bis zu dem (dort) von mir zitierten Jim Al-Khalili. In späteren Auftritten von Sean Carroll sieht man dann, dass er auf einmal Wert darauf legt, dass das Spalten der Welt gar nicht so häufig vorkommt, und in gewisser Hinsicht sogar eher selten ist. Für die verteidigbare Version müsste ich wohl eher David Wallace lesen. (Habe ich jetzt auch versucht ... aha, selbst David Deutsch ist ursprünglich auch schon in diese Falle getappt.)

TomS
02.07.21, 13:41
Interessanter Beitrag.

Für mich persönlich haben die isolierten und unzusammenhängenden Darstellungen von Zeh trotzdem am meisten Überzeugungsarbeit geleistet, VW und Dekohärenz ernst zu nehmen. Er hat halt klar die Probleme aufgezeigt, erklärt wieso die triviale Lösung zu kurz greift, und auch bei der von ihm bevorzugten Lösung nicht die Probleme verschwiegen.
Welche triviale Lösung wäre das?

An Zeh finde ich interessant, dass er ohne Kenntnis der Arbeit von Everett über die Dekohärenz zu einer Variante der MWI gelangt, wobei ihm die Dekohärenz letztlich eine Begründung liefert, die Everett so nicht hatte.

Bei Zeh habe ich zum ersten Mal gelesen, dass man für die VW mehr braucht, als nur einen Hilbertraum und einen Hamiltonian.
Verstehe ich nicht.

Man hat auf fundamentaler Ebene ja tatsächlich nur eine Produkthilbertraum und einen entsprechenden Hamiltonian.

... und festgestellt, dass wohl Sean Carroll in seinem Buch versehentlich die VW Interpretation in einer angreifbaren Version dargestellt hat. (Die Welt wird zu oft, und in zu viele Welten gespalten.)
Interessanter Punkt.

Welches Buch meinst du? Und wie genau unterscheiden sich die Versionen?

Für die verteidigbare Version müsste ich wohl eher David Wallace lesen ...
Harter Tobak ;-)

Jakito
02.07.21, 16:01
Verstehe ich nicht.

Man hat auf fundamentaler Ebene ja tatsächlich nur eine Produkthilbertraum und einen entsprechenden Hamiltonian.

Da gibt es viele Antworten. Zeh selber schrieb: "Of course, no unitary treatment of the time dependence can explain why only one of these dynamically independent components is experienced. Erich Joos and Dieter Zeh,1985 [150, p.242]"
[150] E. Joos and H. D. Zeh, The emergence of classical properties through interaction with the environment, Zeitschrift f. Physik B Condensed Matter 59 (1985), 223–43.
http://www.thp.uni-koeln.de/gravitation/zeh/J+Z.pdf

In dem Thread auf PhysicsForums habe ich aber schon kräftiger hingelangt (https://www.physicsforums.com/threads/are-there-signs-that-any-quantum-interpretation-can-be-proved-or-disproved.1004469/post-6508574) indem ich auf Carl Friedrich von Weizsäckers Kritik verwiesen habe, dass Messungen auf Basis der rein unitären Quantenmechanik erklären zu wollen natürlich zirkulär ist, und auf Jan-Markus Schwindts Ausarbeitung dieses Arguments in Nothing happens in the Universe of the Everett Interpretation (https://arxiv.org/abs/1210.8447) verlinkt habe.

Schwindts Auflösung des Paradoxes ist, dass Argumente auf Basis von Dekoheränz den Hilbertraum in Teilsysteme und Umgebung faktorisieren, und dass die Auszeichnung einer Umgebung eine Struktur zusätzlich zum Hilbertraum und dem Hamiltonian ist. Und es ist noch schlimmer, weil die Umgebung typischerweise auf Basis der Raumzeitstruktur ausgezeichnet wird, und damit implizit die 3+1 dimensionale Struktur (die alltägliche Erfahrungen eben auszeichnet) zurück in die rein unitäre Quantenmechanik bringt.

Aber was Zeh meint geht noch ein wenig tiefer (als Schwindts Auflösung, die original Kritik hat sich schon eher auf den tieferen Teil bezogen). Dieser tiefere Teil wird sogar von Sean Carroll gut verstanden und addressiert, und auch David Wallace widmet diesem Teil sehr viel Zeit und Raum.

TomS
02.07.21, 18:49
Danke.

Das Argument auf Basis einer Raumzeitstruktur ist interessant. Möglicherweise liefert bereits die relativistische Quantenfeldtheorie plus Cluster Decomposition das Gewünschte.

Letztlich weiß man ja, worauf man hinauswill: eine Zerlegung bzgl. einer bestimmten Observablen-Algebra des Subsystems. Dass dies „richtig“ ist, wissen wir aufgrund der Tatsache, dass die Kopenhagener Interpretation funktioniert. Genau wie in der Frage der Wahrscheinlichkeiten geht es „nur“ darum, dies mittels geeigneter Axiome zu motivieren.

Ich werde mal wieder Wallace konsultieren.

Timm
03.07.21, 09:32
Allerdings habe ich mich jetzt entschlossen, diesem Diskussionsforum beizutreten, und in diesem Thread zu schreiben, weil mir aufgefallen ist, dass Timm derselbe ist, der auf PhysicsForums den entsprechenden Thread losgetreten hat, und mit dem ich auch auf BackReAction einen quasi unfertigen Austausch (http://backreaction.blogspot.com/2021/05/what-did-einstein-mean-by-spooky-action.html?showComment=1620766677814#c26020493652 09694346) hatte.

So begegnet man sich wieder. Willkommen bei quanten.de

Zeilinger schreibt in "Einstein's Schleier" sinngemäß, dass ein "mathematisches Hilfsmittel" nicht kollabiert und es damit die Problematik einer sich instantan auf einen Punkt zusammen ziehenden realen Welle nicht gibt.
"Es gibt keinerlei Notwendigkeit für die Annahme, dass sich die Wellenfunktion tatsächlich im Raum ausbreitet."

TomS
03.07.21, 09:58
Zeilinger schreibt in "Einstein's Schleier" sinngemäß, dass ein "mathematisches Hilfsmittel" nicht kollabiert und es damit die Problematik einer sich instantan auf einen Punkt zusammen ziehenden realen Welle nicht gibt.
"Es gibt keinerlei Notwendigkeit für die Annahme, dass sich die Wellenfunktion tatsächlich im Raum ausbreitet."
Damit löst Zeilinger das Problem einer vermeintlichen Inkonsistenz, zu der ein Kollaps im Rahmen einer ontischen Interpretation der Wellenfunktion führt, durch Verzicht auf die ontische Interpretation.

Das ist klar.

Das war hier jedoch nie mein Punkt, weil ich diese ontische Interpretation hier nie vorausgesetzt habe.

Man erkennt jedoch anhand der oben dargestellten Argumente, dass das Projektionspostulat als rein mathematisches Werkzeug auch in nicht-ontischen Interpretationen zu Problemen führt:
1) in seiner einfachen Form führt es zu explizit falschen Vorhersagen; siehe z.B. die Nebelkammer, aber auch andere Experimente
2) in „verbesserter“ Form ist es nicht axiomatisch formulierbar sondern folgt erst je Einzelsystem nach expliziter Berechnung; es erscheint dann ex post, ohne Vorhersagekraft und ist verzichtbar.

Daher ist die Quantenmechanik ohne Projektionspostulat formulierbar, solange niemand ein explizites Gegenbeispiel findet, in dem experimentell gezeigt wird, dass ohne Projektionspostulat falsche Vorhersagen folgen. Ich kenne kein derartiges Beispiel.

Timm
03.07.21, 11:00
Daher ist die Quantenmechanik ohne Projektionspostulat formulierbar, solange niemand ein explizites Gegenbeispiel findet, in dem experimentell gezeigt wird, dass ohne Projektionspostulat falsche Vorhersagen folgen. Ich kenne kein derartiges Beispiel.
Es geht doch beim Projektionspostulat um das, was man beobachtet. Insofern sollte es unabhängig von jeder Interpretation sein. Was meinst du mit "falschen Vorhersagen"? Abweichungen von der Born'schen Wahrscheinlichkeit?

Zur Klärung, stimmst du mit dieser Definition überein?:

https://www.thefreedictionary.com/projection+postulate

projection postulate

The postulate in quantum mechanics that observation of a physical system, by determining the value of an observable, results in the transition of the quantum state of the system to a particular eigenstate corresponding to the eigenvalue of the observed quantity.

Oder mit PeterDonis im erwähnten Thread?:

In the basic math of QM, this is simply put in by hand as the projection postulate: there is nothing anywhere else in the math that predicts it.

The only way to get a single definite outcome out of that in the basic math is to put in the projection postulate by hand--i.e., just declare by fiat that we collapse the wave function in the math whenever we have to to make predictions for future experimental results come out right.

On a collapse interpretation, the projection postulate becomes an actual physical law and wave function collapses become actual physical events

On a no collapse interpretation like the MWI, the projection postulate remains just a mathematical device, and the rationale for applying it is that, once decoherence happens after a measurement, the different branches of the wave function can never interact with each other again, so in each individual branch we can apply the projection postulate to get an "effective" wave function for that branch that works for predicting future measurement results in that branch, even though we "know" (if we accept the MWI as true) that there are other branches in the overall wave function.

TomS
03.07.21, 11:31
Es geht doch beim Projektionspostulat um das, was man beobachtet. Insofern sollte es unabhängig von jeder Interpretation sein. Was meinst du mit "falschen Vorhersagen"?
Nein, das Projektionspostulat alleine sagt nichts über eine einzelne Beobachtung aus!

In der von dir zitierten Formulierung

The postulate in quantum mechanics that observation of a system, by determining the value of an observable, results in the transition of the quantum state of the system to a particular eigenstate corresponding to the eigenvalue of the observed quantity

führt es in vielen Fällen zu explizit falschen Vorhersagen. Das habe ich oben mehrfach anhand der Nebelkammer erklärt; das ist vielen Physikern, die sich ernsthaft mit der Materie befassen klar, das war schon in den zwanziger Jahren klar (was andere Physiker nicht daran hindert, diese falsche Formulierung bis heute weiter zu verbreiten - aber das Fass machen wir jetzt nicht auch noch auf).

Waren meine Erklärungen wirklich so unverständlich? Muss ich nochmal ganz von vorne anfangen?

Ok, Vorschlag: Ich erkläre, warum das Projektionspostulat in keiner bekannten Fassung irgendeine beobachtbare Konsequenz für eine einzelne Messung hat. Ich erkläre, warum es in der von dir zitierten Fassung für die Nebelkammer explizit falsche Vorhersagen macht. Ich erkläre, warum man anhand von Mott’s Arbeit erkennt, dass das Projektionspostulat in jeder beliebigen Fassung für den Fall der Nebelkammer irrelevant ist. Ich erkläre, wie man die oben zitierte Fassung so modifizieren kann, so dass zutreffende Vorhersagen folgen, jedoch zugleich die Irrelevanz des Projektionspostulates als fundamentales Postulat. Ich erkläre, wie daraus ontische oder nicht-ontische Kollaps-freie Interpretationen der Quantenmechanik (d.h. ohne Projektionspostulat) folgen, die experimentell überprüft werden können, wobei der explizite Verzicht auf das Projektionspostulat ein Gegenstand dieser Prüfung ist.

Wenn du gestattest, würde ich das gerne anhand des Mott-Problems erläutern. Sobald das verstanden ist, können wir diskutieren, dass und warum das verallgemeinerbar ist.

Und du versprichst mir bitte, dass du nicht zwischendrin Quellen heranziehst, die unvollständig oder fehlerhaft sind und nur unsere Zeit verschwenden. Bitte! Nur meine Darstellung verstehen und diese gerne kritisch hinterfragen. Gerne auch später mit anderen Darstellungen kritisch vergleichen, aber erst, wenn alles verstanden ist, sonst ist wieder alles vergebens.

Ok?

Ich muss dazu ein paar Formeln benutzen, daher verlinke ich ein PDF.

Timm
03.07.21, 13:56
Ich nehme dir ja ab, dass die Nebelkammer eine Ausnahme darstellt.

Du musst nicht nochmal von vorne anfangen.

TomS
03.07.21, 14:07
Die Nebelkammer stellt keine Ausnahme dar. Wie kommst du zu dieser Idee? Wer sagt das?

Irgendwie habe ich den Eindruck, dass du die Argumentation nicht wirklich verstehst.

Ich schreibe das trotzdem mal zusammen, schon alleine um diese Gedanken mal zu konsolidieren und von diesen zusammengesucht Zitaten wegzukommen; kann man immer wieder brauchen.

Timm
03.07.21, 15:23
Du hast mich falsch verstanden. Es gibt Ausnahmen, bei welchen die von mir zitierte Definition des Projektionspostulats "zu falschen Vorhersagen führt".

TomS
03.07.21, 15:54
Du hast mich falsch verstanden. Es gibt Ausnahmen, bei welchen die von mir zitierte Definition des Projektionspostulats "zu falschen Vorhersagen führt".
Wenn das so ist, dann ist es offenbar als fundamentales Postulat - und als solches wird es leider häufig genannt - untauglich.


Ich nenne mal eine ähnlich absurde Vorgehensweise aus der Elektrodynamik.

Die Elektrodynamik beruht auf den Maxwellschen Gleichungen sowie der Kontinuitätsgleichung für elektrische Ladungen und Ströme. In dieser Form ist die Theorie sowohl vollständig als auch mathematisch konsistent.
Nun ist es in vielen Fällen jedoch zu kompliziert, mit dieser fundamentalen Darstellung zu rechnen. Man führt zum Beispiel effektive Gleichungen ein, die in makroskopischen Systemen nützlich sind. Ein Beispiel ist der Zusammenhang zwischen elektrischer Stromdichte j, der Leitfähigkeit σ und dem elektrischem Feld E:

j = σE

Kann man das o.g. System aus Maxwellschen Gleichungen und Kontinuitätsgleichung nun um diese Gleichung ergänzen?

Nein, denn der Zusammenhang gilt nur in Spezialfällen und ist für einen Supraleiters explizit und nachweislich falsch; hier gelten die London-Gleichungen.